Tải bản đầy đủ (.pdf) (65 trang)

Chim cu thay doi hinh dang

Bạn đang xem bản rút gọn của tài liệu. Xem và tải ngay bản đầy đủ của tài liệu tại đây (1.64 MB, 65 trang )

<span class='text_page_counter'>(1)</span><div class='page_container' data-page=1>

<b>TUYỂN TẬP ĐỀ THI HỌC SINH</b>


<b>GIỎI MÔN HĨA HỌC</b>



[Tài liệu tổng hợp từ các đề thi HSG]


[copyright©2010 />YDS


</div>
<span class='text_page_counter'>(2)</span><div class='page_container' data-page=2>

<b>PHẦN 1: CÁC ĐỀ THI HỌC SINH GIỎI THÀNH PHỐ ĐÀ NẴNG</b>



(Nguồn: Thầy Đặng Công Anh Tuấn – GV Trường THPT Chuyên Lê Quý Đôn – Đà Nẵng)


<b>ĐỀ SỐ 1</b>


<b>SỞGIÁO DỤC VÀ ĐÀO TẠO</b>
<b>THÀNH PHỐ ĐÀ NẴNG</b>


<b>HƯỚNG DẪN CHẤM ĐỀTHI HỌC SINH GIỎI THÀNH PHỐ</b>


<b>NĂM HỌC 2007 - 2008</b>
<b>MƠN: HĨA HỌC LỚP 1O</b>


Thời gian: 150 phút (khơng kểthời gian giao đề)


<b>Câu I</b> (4 điểm)


X và Y là các nguyên tố nhóm A, đều tạo hợp chất với hiđro có dạng RH (R là kí hiệu của ngun tốX hoặc Y).
Gọi A và B lần lượt là hiđroxit ứng với hóa trịcao nhất của X và Y. Trong B, Y chiếm 35,323% khối lượng. Trung
hịa hồn tồn 50 gam dung dịch A 16,8% cần 150 mL dung dịch B 1M.


<b>1.</b> Xác định các nguyên tốX và Y.


<b>2.</b> B’ là anion tương ứng của phân tửB. (a) Hãy cho biết (có cơng thức minh họa) dạng hình học của B và B’. (b)


So sánh (có giải thích) độdài liên kết Y-O trong phân tử B và B’.


<b>3. Bi</b>ết X có cấu trúc tinh thểlập phương tâm khối và mật độsắp xếp tương đối được định nghĩa bằng tỉ lệgiữa
thểtích chiếm bởi các hình cầu trong tế bào cơ sởvà thểtích tế bào cơ sở. Hãy tính mật độsắp xếp tương đối
trong tinh thểcủa X.


<b>ĐÁP ÁN</b> <b>ĐIỂM</b>


<b>1. H</b>ợp chất với hiđro có dạng RH nên Y có thểthuộc nhóm IA hoặc VIIA.
Trường hợp : Nếu Y thuộc nhóm IA thì B có dạng YOH


Ta có : Y 9,284


677
,
64


323
,
35
17


Y <sub></sub> <sub></sub> <sub></sub>


(loại do khơng có nghiệm thích hợp)


Trường hợp 2 : Y thuộc nhóm VIIA thì B có dạng HYO4


Ta có : Y 35,5



677
,
64


323
,
35
65


Y <sub></sub> <sub></sub> <sub></sub>


, vậy Y là nguyên tốclo (Cl).


B (HClO4) là một axit, nên A là một bazơ dạng XOH


gam
4
,
8
gam
50
100


8
,
16


m<sub>A</sub>   


XOH + HClO4XClO4+ H2O



 n n 0,15L 1mol/L 0,15mol
4


HClO


A    




mol
15
,
0


gam
4
,
8
mol
/
gam
17


M<sub>X</sub>  


</div>
<span class='text_page_counter'>(3)</span><div class='page_container' data-page=3>

(a) Dạng hình học :


Axit pecloric



(dạng tứdiện)


Ion peclorat


(dạng tứdiện đều)


(b) Bậc liên kết càng lớn độdài liên kết càng nhỏ, do vậy :


<b>3. G</b>ọi a là độdài cạnh ô mạng cơ sởvà R là bán kính nguyên tử, ta có 3a4R


Sốngun tửcó trong một ơ mạng cơ sởbằng : 1 2
8
1
8  


Vậy 68%


a
R
3
4
2
f


3
3


v 







<b>Câu II</b> (4 điểm)


<b>1. Cho bi</b>ết số oxi hóa của mỗi nguyên tử lưu huỳnh (S) trong phân tử axit thiosunfuric (H2S2O3) và của mỗi


nguyên tửcacbon trong phân tửaxit axetic (CH3COOH)


<b>2.</b> Thêm lượng dư dung dịch KI (có pha hồtinh bột) vào 5,00 mL dung dịch K2Cr2O7 có nồngđộ a M trong


H2SO4, thì dung dịch thu được có màu xanh. Thêm tiếp dung dịch Na2S2O30,10 M vào cho đến khi màu xanh


biến mất thìđã dùng 15,00 mL dung dịch này. Viết các phương trình phảnứng và tính a. Biết sản phẩm oxi hóa
S2O32-là S4O62-.


<b>3. Hịa tan hồn tồn 9,06 gam m</b>ột mẫu hợp kim Al-Mg (giảthiết khơng có tạp chất nào khác) bằng dung dịch
H2SO4đặc, thu được 12,22 L khí SO2(đo ở136,5oC; 1,1 atm) và 0,64 gam chất rắn màu vàng. Xác định phần


</div>
<span class='text_page_counter'>(4)</span><div class='page_container' data-page=4>

<b>ĐÁPÁN</b> <b>ĐIỂM</b>
<b>1. S</b>ốoxi hóa của các nguyên tửS và C :


<b>1.0 đ</b>


<b>2.</b> Phương trình phảnứng :


6KI + K2Cr2O7+ 7H2SO43I2+ Cr2(SO4)3+ 4K2SO4+ 7H2O (1)


2Na2S2O3+ I22NaI + Na2S4O6 (2)



<b>0,5 đ</b>


Từ(1) và (2) ta có : 0,015L 0,1mol/L 2,5.10 mol
6


1
n


6
1


n<sub>K</sub> <sub>Cr</sub><sub>O</sub> <sub>Na</sub><sub>S</sub><sub>O</sub> 4


3
2
2
7
2
2







 0,05M


L
10


.
5
mol
10
.
5
,
2
a
3
4

 <sub></sub>
<b>0,5 đ</b>


<b>3. G</b>ọi x, y lần lượt là sốmol các kim loại Mg và Al.


)
mol
(
4
,
0
5
,
1
273
273
4
,


22
22
,
12
1
,
1
n
2
SO 




 ; Chất rắn là S, 0,02(mol)


32
64
,
0
n<sub>S</sub>  


0
02
.
0
12
,
0
6


y
4
4
,
0
8
,
0
6
y
3
3
0
y
x
2
2
0
x
S
e
6
S
S
e
2
S
e
3
Al

Al
e
2
Mg
Mg













<b>1,5 đ</b>


Ta có :




 <sub></sub> <sub></sub> <sub></sub>




22


,
0
y
;
13
,
0
x
92
,
0
y
3
x
2
06
,
9
y
27
x
24


 100% 34,44%


gam
06
,
9
mol


/
gam
24
mol
13
,
0
m


% <sub>Mg</sub>     và %m<sub>Al</sub> 65,56%


<b>0,5 đ</b>


<b>Câu III</b>(4 điểm)


<b>1.</b> Nguyên tửcủa nguyên tốX có tổng sốhạt các loại là 60, sốhạt mang điện trong hạt nhân bằng sốhạt khơng mang
điện; Ngun tửcủa ngun tốY có 11 electron p ; nguyên tửnguyên tốZ có 4 lớp electron và 6 electron độc thân.


a)

Dựa trên cấu hình electron, cho biết vịtrí của các ngun tốtrên trong bảng hệthống tuần hồn.


b)

So sánh (có giải thích) bán kính của các ngun tửvà ion X, X2+và Y-.


</div>
<span class='text_page_counter'>(5)</span><div class='page_container' data-page=5>

<b>3. S</b>ục Cl2vào dung dịch KOH lỗng thuđược dung dịch A, hịa tan I2vào dung dịch KOH lỗng thuđược dung


dịch B (tiến hànhởnhiệt độphịng).


a) Viết phương trình hóa học xảy ra và cho nhận xét.


b) Nêu hiện tượng và viết phương trình hóa học xảy ra khi cho lần lượt các dung dịch hỗn hợp HCl và FeCl2,



dung dịch Br2, H2O2vào dung dịch A (khơng có Cl2dư).


<b>ĐÁP ÁN</b> <b>ĐIỂM</b>


<b>1.</b> a) Xác định vịtrí dựa vào cấu hình electron:


X X X X X


2Z N 60 ; Z N Z 20, X là canxi (Ca), cấu hình electron của20Ca:[Ar] 4s2


Y có 11 electron p nên cấu hình của Y là 1s22s22p63s23p5hay [Ne] 3s23p5Y là Cl
Theo giảthiết thì Z chính là crom, cấu hình electron của24Cr: [Ar] 3d54s1


STT Chu kỳnguyên tố Nhóm nguyên tố


Ca 20 4 IIA


Cl 17 3 VIIA


Cr 24 4 VIB


Trật tự tăng dần bán kính nguyên tử: R<sub>Ca</sub>2 R<sub>Cl</sub> R<sub>Ca</sub>


b) Bán kính nguyên tửtỉlệvới thuận với sốlớp electron và tỉlệnghịch với số đơn vị điện
tích hạt nhân của ngun tử đó. Bán kính ion Ca2+nhỏ hơn Cl-do có cùng số lớp electron (n
= 3), nhưng điện tích hạt nhân Ca2+(Z = 20) lớn hơn Cl-(Z = 17). Bán kính ngun tửCa lớn
nhất do có sốlớp electron lớn nhất (n = 4).


<b>2,0 đ</b>



<b>2.</b> Điều chếHCl từdung dịch H2SO4đậm đặc và NaCl rắn, hình 5.5 trang 128 SGK Hóa học 10


nâng cao.


<b>0,75 đ</b>


<b>3.</b> a)Ởnhiệt độ thường:


2KOH + Cl2KCl + KClO + H2O


6KOH + 3I25KI + KIO3+ 3H2O


Giải thích: Trong mơi trường kiềm tồn tại cân bằng: 3XO-⇌X-+ XO<sub>3</sub>


Ion ClO-phân hủy rất chậmởnhiệt độ thường và phân hủy nhanh khi đun nóng, ion IO-phân
hủyởtất cảcác nhiệt độ.


b) Các phương trình hóa học:


Ion ClO-có tính oxi hóa rất mạnh, thểhiện trong các phương trình hóa học:


- Khi cho dung dịch FeCl2và HCl vào dung dịch A: có khí vàng lục thốt ra và dung dịch từ


</div>
<span class='text_page_counter'>(6)</span><div class='page_container' data-page=6>

không màu chuyển sang màu vàng nâu:


2 FeCl2+ KClO + 2HCl2 FeCl3+ Cl2 + H2O


- Khi cho dung dịch Br2vào dung dịch A : dung dịch brom mất màu :


Br2+ 5KClO + H2O2HBrO3+ 5KCl



- Khi cho H2O2vào dung dịch A: có khí khơng màu, khơng mùi thoát ra:


H2O2+ KClOH2O + O2+ KCl
<b>Câu IV</b>(4 điểm)


<b>1</b> Cho biết: năng lượng liên kết của các liên kết H-H, O-O, O=O, H-O lần lượt là 436, 142, 499, 460 ( kJ/mol).
Hãy viết phương trình nhiệt hóa học của phảnứng giữa khí hiđro và khí oxi tạo ra hiđropeoxit.


<b>2</b> Cho phảnứng: 2SO2(k) + O2(k)  2SO3(k) H = - 198 kJ


a)

Để tăng hiệu suất quá trình tổng hợp SO3, người ta có thểsửdụng biện pháp nào liên quan đến áp suất, nhiệt độ


và chất xúc tác? Giải thích?


b)

Cho 10,51 mol khí SO2và 37,17 mol khơng khí (20% vềthểtích là O2cịn lại là N2) có xúc tác là V2O5. Thực


hiện phảnứngở4270C, 1 atm thì phảnứng đạt hiệu suất 98%. Tính hằng sốcân bằng KC, KPcủa phảnứngở


4270C.


<b>3</b> A là một oxit của sắt. Lấy một lượng A chia làm 2 phần bằng nhau. Phần I tác dụng vừa đủvới a mol H2SO4


trong dung dịch H2SO4loãng. Phần II tác dụng vừa đủvới b mol H2SO4trong dung dịch H2SO4đặc nóng tạo


SO2(sản phẩm khửduy nhất). Biết b = 1,25a, xác định công thức oxit sắt ban đầu.


<b>ĐÁP ÁN</b> <b>ĐIỂM</b>


<b>1.</b> Hpư= EH-H+ EO=O- 2EO-H - EO-O


Hpư= 436 + 499 -2.460–142 = -127 (kJ)


H2(k) + O2(k)  H2O2(k) H = - 127 kJ


<b>1,0 đ</b>


- Giảm nhiệt độcủa hệphảnứng, nhưng khoảng 5000C là thích hợp vì: nếu giảm thấp q thì
tốc độphảnứng chậm.


- Thổi liên tục SO2và khơng khí được nénởáp suất cao vào lị phảnứng vì phản ứng xảy ra


theo chiều thuận làm giảm áp suất của hệ.


- Dùng V2O5làm xúc tác đểphảnứng mau chóng đạt trạng thái cân bằng.


<b>0,75 đ</b>


<b>2. nO</b>2 bđ= 7,434 (mol), nN2 bđ= 29,736 (mol)


2SO2 (k) + O2  2SO3(k) H = - 198 kJ


Ban đầu: 10,51 (mol) 7,434 (mol) 0
Lúc phảnứng: 10,3 (mol) 5,15 (mol) 10,3 (mol)
Lúc CB: <b>0,21 (mol) 2,284 (mol) 10,3 (mol)</b>


∑sốmol hỗn hợpởTTCB = 0,21 + 2,284 + 10,3 + 29,736 = 42,53 (mol)
Pi= xi.P = xi.1 = xi


</div>
<span class='text_page_counter'>(7)</span><div class='page_container' data-page=7>

2
3



P 2


2 2


(Pso )
K =


(Pso ) .Po và


- n


C P


K =K (RT)


(R = 0,082, T = 427 + 273 = 7000K,n = -1)


2


4


P 2


(10,3) .42,53


K = >> 4,48.10


(0,21) .2,284



 và K =4,48.10 .(0,082.700)<sub>C</sub> 4 -(-1) 257.10 4


<b>3. G</b>ọi FexOylà công thức của A


( 1) 2FexOy + 2yH2SO4  xFe2(SO4)2y/x + 2y H2O


n ny


(2) 2FexOy + (6x-2y) H2SO4  xFe2(SO4)3+ (3x-2y) SO2+ ( 6x-2y) H2O


n n (3x-y)


Theo giảthiết : n(3x-y) = 1,25 ny x 2, 25 3


y  3  4 A là Fe3O4


<b>1,0 đ</b>


<b>Câu V</b> ( 4 điểm)


<b>1.</b> Từdung dịch H2SO498% ( D= 1,84 g.mL
-1


) và dung dịch HCl 5 M, trình bày phương pháp pha chế để được
200 mL dung dịch hỗn hợp H2SO41M và HCl 1M .


<b>2.</b> Đốt cháy hoàn toàn a gam S rồi cho sản phẩm sục qua 200 mL dung dịch NaOH b M thu được dung dịch X.
Chia X làm hai phần bằng nhau. Phần 1 cho tác dụng với dung dịch CaCl2dư thấy xuất hiện c gam kết tủa .


Phần 2 tác dụng với dung dịch nước vôi dư thấy xuất hiện d gam kết tủa . Biết d >c. Tìm biểu thức quan hệ


giữa a và b.


<b>3.</b> Cho các dung dịch riêng biệt mất nhãn sau: Na2SO4, AlCl3, FeSO4, NaHSO4, FeCl3. Một học sinh cho rằng nếu


dùng dung dịch Na2S thì có thểphân biệt các dung dịch trên ngayởlần thử đầu tiên. Kết luận của học sinh đó


có đúng khơng ? Vì sao?


<b>ĐÁP ÁN</b> <b>ĐIỂM</b>


<b>1.</b>


* Phần tính tốn :


Sốmol H2SO4cần lấy = sốmol HCl cần lấy =


200 1


0, 2 (mol)
1000


 <sub></sub>


Khối lượng dung dịch H2SO4cần lấy : mdd=


0, 2 98 100


20 (gam)
98



  <sub></sub>


Thểtích dung dịch H2SO4 cần lấy = 20 : 1,84 = 10,87 (mL)


Thểtích dung dịch HCl cần lấy = 0,2 : 5 = 0,04 (L) = 40 mL


</div>
<span class='text_page_counter'>(8)</span><div class='page_container' data-page=8>

Cân 20 gam dung dịch H2SO4hoặc đong 10,87 mL dung dịch H2SO4, sau đó cho từtừ vào


bình chứa nước khuấy đều. Đợi dung dịch H2SO4thật nguội, đong 40 mL dung dịch HCl 5M


thêm vào bình, sauđó thêm nước vào cho đến vạch 200 mL


<b>2.</b>Phương trình :
(1) S + O2SO2


(2) SO2+ NaOH NaHSO3


(3) SO2+ 2 NaOHNa2SO3+ H2O


Phần I tác dụng với dung dịch CaCl2sinh kết tủa, chứng tỏdung dịch X có chứa Na2SO3, phần


II tác dụng với dung dịch Ca(OH)2 sinh nhiều kết tủa hơn chứng tỏ dung dịch X có muối


NaHSO3


(4) Na2SO3+ CaCl2CaSO3+ 2NaCl


(5) NaHSO3+ Ca(OH)2CaSO3+ Na2SO3+ 2H2O


ns= a/32 (mol) , nNaOH= 0,2 b ( mol)



Theo (2),(3), đểSO2tác dụng với dung dịch NaOH sinh 2 loại muối thì :


2


NaOH NaOH
SO S


n n


1 2


n n


    1 <0, 2b 6, 4b


a a


32


 < 2


Vậy : a b a


6, 4 3, 2


<b>1,5 đ</b>


<b>3. K</b>ết luận của học sinh trên đúng, vì khi cho dung dịch Na2S lần lượt vào mẫu thử của các



dung dịch trên thì:


<b>-</b> Mẫu thửkhơng có hiện tượng chứa dung dịch Na2SO4


<b>-</b> Mẫu thửxuất hiện kết tủa trắng keo và có hiện tượng sủi bọt khí chứa AlCl3: 2AlCl3+ 3


Na2S + 3H2O6NaCl + 2Al(OH)3+ 3H2S


<b>-</b> Mẫu thửcó hiện ttượng sủi bọt khí chứa dung dịch NaHSO4


2 NaHSO4+ Na2S2Na2SO4+ H2S


Mẫu thửxuất hiện kết tủa đen chứa FeCl2: Na2S + FeCl2FeS + 2NaCl
<b>-</b> Mẫu thửxuất hiện kết tủa đen và vàng có chứa FeCl3


2FeCl3+ 3Na2S6NaCl + S + 2FeS


</div>
<span class='text_page_counter'>(9)</span><div class='page_container' data-page=9>

ĐỀ SỐ 2


<b>SỞGIÁO DỤC VÀ ĐÀO TẠO</b>
<b>THÀNH PHỐ ĐÀ NẴNG</b>


<b>KÌ THI CHỌN HỌC SINH GIỎI LỚP 10</b>


<b>NĂM HỌC 2008-2009</b>


<b>MƠN THI: HĨA HỌC</b>


Thời gian làm bài : 150 phút (khơng kểthời gian giao đề)



<b>ĐỀCHÍNH THỨC</b>


<i>(Đềnày gồm có hai trang)</i>


<b>Câu I:</b>( 2,0 điểm)


<b>1.</b> (a)X,Y đều là hai nguyên tốnhóm A. Nguyên tửX có tổng sốelectron s là 7, Y là nguyên tốp, có sốelectron
lớn hơn 15, có 3 lớp electron và có 2 electron p độc thânởtrạng thái cơ bản. Xác định X, Y và gọi tên. (b) So
sánh bán kính của ion A+, B2-và giải thích. (c) Viết sơ đồhình thành liên kết trong oxit bậc cao nhất của X. Viết
công thức cấu tạo và cho biết dạng hình học của oxit bậc cao nhất và hiđroxit tương ứng của Y.


<b>2.</b> (a)Xác định x, y và hồn thành phươngtrình hạt nhân 238<sub>92</sub>U 206<sub>82</sub>Pb x He<sub>2</sub>4 y e<sub></sub>0<sub>1</sub>


(b) Một mẫu đá chứa 35 mg 238<sub>92</sub>Uvà 3 mg 206<sub>82</sub>Pb<b>. Tính th</b>ời gian tồn tại của mẫu đá đó, biết chu kì bán hủy của


238


92U là 4,51.10
9


năm.


<b>3.</b> M là một kim loại có khối lượng nguyên tửlà 1,0550817.10-22gam. Trong tự nhiên M có hai đồng vị hơn kém
nhau 2 nơtron. Tỉlệ% sốnguyên tửcủa đồng vịnhẹ hơn trong tự nhiên là 73%. Xác định sốkhối hai đồng vị
và tính % vềkhối lượng của đồng vịnhẹtrong oxit MO. (Cho: 1u = 1,6605.10-24gam, O = 16, giá trịnguyên tử
khối của mỗi đồng vịbằng sốkhối.)


<b>Câu II:</b>( 2,0 điểm)


<b>1.</b> a) Xác định sốoxi hoá của P, S, Pb trong các chất sau : POCl3; Na2S2O3; Pb3O4.


<b>2.</b> Hoàn thành và cân bằng các phảnứng sau theo phương phápcân bằng electron:


a) Fe3O4 + HIFeI2 + I2 + ?


b) CrI3+ KOH + Cl2K2CrO4+ KIO4+ ? + ?


<b>3.</b> Hòa tan 1,0 gam một quặng sắt chứa Fe2O3và tạp chất trơ trong dung dịch HCl dư, loại tạp chất, thu được dung


dịch A. Dung dịch A phảnứng vừa đủ với một lượng dung dịch KI thu được dung dịch B và chất rắn C. Chất
rắn C tác dụng vừa đủvới 25 ml dung dịch Na2S2O30,2 M. Tính % khối lượng Fe2O3 trong quặng sắt nói trên.


Cho các phảnứng xảy ra hoàn toàn.


<b>Câu III:</b>( 2,0 điểm)


<b>1.</b> a) Viết phương trình hóa học thực hiện chuyển hóa sau, ghi rõđiều kiện phảnứng.


1 2 3 4


3 2 2 3


</div>
<span class='text_page_counter'>(10)</span><div class='page_container' data-page=10>

b) (SCN)2là một halogen giả. Biết tính oxi hóa của Br2> (SCN)2> I2. Viết phương trình hóa học minh họa kết


quảso sánh trên .


<b>2.</b> Tại sao có sựkhác biệt vềgóc liên kết của Cl2O (1110) và OF2(1050) ?


<b>3.</b> Để tách brom có trong 1m3 nước biển dưới dạng NaBr, người ta cho một lượng dung dịch H2SO4 vào một


lượng nước biển; tiếp đến sục khí clo vào dung dịch mới thu được; sau đó dùng khơng khí lơi cuốn hơi brom


vào dung dịch Na2CO3tới khi bão hoà brom. Cuối cùng cho H2SO4vào dung dịch đã bão hoà brom, thu hơi


brom rồi hố lỏng, được 28,05 ml Br2có khối lượng riêng 3,12 g/mlở200C. Viết phương trình hố học chủyếu


xảy ra trong q trìnhđó và cho biết vai trị của H2SO4. Tính % khối lượng của brom trong nước biển biết khối


lượng riêng của nước biển là 1,25 g/ml.


<b>Câu IV:</b>( 2,0 điểm)


<b>1.</b> Cho hỗn hợp Fe và FeS tác dụng với dung dịch H2SO4đậm đặc nóng thu được dung dịch X làm mất màu dung


dịch K2Cr2O7trong H2SO4lỗng. Viết phương trình hóa học của các q trình thí nghiệm trên.


<b>2.</b> A là hợp chất của lưu huỳnh, tan rất tốt trong nước tạo dung dịch X chứa một chất tan. Hòa tan 25,8 gam A vào
nước thu được 200 ml dung dịch X ( D = 1,15 g/ml). Chia dung dịch X làm hai phần bằng nhau. Phần 1 trung hòa
vừa đủvới 100 ml dung dịch NaOH 10% (D = 1,2 g/ml). Phần 2 tác dụng với dung dịch BaCl2dư thu được một


kết tủa trắng không tan trong axit. Xác định công thức phân tửvà viết công thức cấu tạo của A.


<b>Câu V:</b>(2,0 điểm)


<b>1.</b> Hằng sốcân bằng KCở250C của phảnứng Cl2(k) ⇌2Cl (k) là 1,4.10-18 . Tính nồng độClở250C lúc cân


bằng biết nồng độcủa Cl2lúc cân bằng là 0,896 M. Từ đó cho biếtở250C, clo tồn tại chủyếuởdạng phân tử


hay nguyên tử?


<b>2.</b> Cho biết cân bằng 2FeCl2 (aq) + Cl2 (aq) ⇌ FeCl3 (aq)sẽchuyển dời theo chiều nào khi sục một lượng khí H2S



thích hợp vào dung dịch? ChoE 0,77V,E 1,36V
Cl


2
Cl
0
Fe


Fe
0


2
2


3  







.


<b>3.</b> Phảnứng chuyển hoá một loại kháng sinh trong cơ thể ngườiởnhiệt độ370C có hằng sốtốc độbằng 4,2.10-5
(s-1). Việc điều trị bằng loại kháng sinh trên chỉ có kết quảnếu hàm lượng kháng sinh ln ln lớn hơn 2,00
mg trên 1,00 kg trọng lượng cơ thể. Một bệnh nhân nặng 58 kg uống mỗi lần một viên thuốc chứa 300 mg
kháng sinh đó.


a) Hỏi bậc của phảnứng chuyển hoá?



b) Khoảng thời gian giữa 2 lần uống thuốc kếtiếp là bao lâu?


c) Khi bệnh nhân sốt đến 38,50C thì khoảng cách giữa 2 lần uống thuốc thay đổi như thếnào? Biết năng
lượng hoạt hoá của phảnứng bằng93,322 kJ.mol1.


---HẾT


<i>---Chú ý: Họcsinh được sửdụng Hệthống tuần hoàn các ngun tốhóa học và máy tính cá nhân đơn giản theo quy</i>


</div>
<span class='text_page_counter'>(11)</span><div class='page_container' data-page=11>

<b>ĐỀ SỐ 3</b>


<b>SỞ GIÁO DỤC VÀ ĐÀO TẠO</b>
<b>THÀNH PHỐ ĐÀ NẴNG</b>


<b>HƯỚNG DẪN CHẤM KÌ THI HỌC SINH GIỎI THÀNH PHỐ</b>
<b>NĂM HỌC 2004 - 2005</b>


<b>MƠN: HĨA HỌC LỚP 11</b>


Thời gian: 150 phút (không kể thời gian giao đề)


<b>CÂU I</b> (3 điểm)


1. Hãy so sánh và giải thích sự khác nhau về độ phân cực phân tử, nhiệt độ sơi và độ mạnh tính bazơ giữa NH3và
NF3.


2. N2O4phân li 20,0% thành NO2ở 27oC và 1,00 atm. Hãy xác định (a) giá trị Kp; (b) độ phân li của N2O4tại 27oC
và 0,10 atm; (c) độ phân li của 69g N2O4trong bình 20 L ở 27oC.


3. Tính pH của dung dịch thu được khi thổi hết 224 mL khí CO2vào 200 mL dung dịch NaOH 0,05M, biết axit


cacbonic có pK<sub>a</sub><sub>1</sub> 6,35, pK<sub>a</sub><sub>2</sub> 10,33.


ĐÁP ÁN ĐIỂM


1. Cấu tạo:


N
H H
H
N
F F
F


<b>-</b> NH3phân cực hơn NF3do trong NH3lưỡng cực liên kết và lưỡng cực electron tự do cùng
chiều, còn trong NF3lưỡng cực liên kết và lưỡng cực electron tự do ngược chiều.


<b>-</b> Nhiệt độ sôi của NH3cao hơn do NH3tạo được liên kết H liên phân tử.


<b>-</b> NH3là một bazơ cịn NF3thì không, do trong NF3các nguyên tử F hút electron làm giảm
mật độ electron trên nguyên tử N.


2. Xét phản ứng phân li:


N2O4  2NO2


n 0


n 2n


n-n 2n



Phần mol:




1
1



1
2
, P
1
4
P
P
P
K <sub>2</sub>
2
O
N
2
NO
O
N
2
NO
P


4
2
2
4
2
2 










(a) 1 0,17


)
2
,
0
(
1
)
2
,
0
(
4


P
1
4
K <sub>2</sub>
2
2
2


P <sub></sub>  










(b) 0,10 0,17 0,546 (54,6%)
1
4
2
2










(c) 0,75mol
92


69
n 


</div>
<span class='text_page_counter'>(12)</span><div class='page_container' data-page=12>

)
1
(
9225
,
0
20
300
082
,
0
)
1
(
75
,
0
P
4
2O


N  














 1,845


20
300
082
,
0
.
75
,
0
.
2
P
2
NO
17
,


0
)
1
(
9225
,
0
)
845
,
1
(
K
2
P <sub></sub><sub></sub> 




  0,1927 (19,27%)


3. 0,01mol, n 0,2 0,05 0,01


4
,
22
224
,
0


n<sub>CO</sub> <sub>NaOH</sub>



2     


Vì số mol CO2và NaOH bằng nhau nên hệ chỉ chứa NaHCO3. Có thể tính pH của hệ lưỡng tính
này bằng cơng thức:


6,35 10,33

8,3
2
1
)
pK
pK
(
2
1


pH  <sub>1</sub> <sub>2</sub>   


<b>1,50</b>
(0,503)


<b>0,75</b>
(0,25+0,5)


<b>CÂU II</b> (3 điểm)


2. Viết phương trình phản ứng xảy ra khi cho NH4Cl tác dụng với CuO và với ZnO. Cho biết ứng dụng thực tế của
NH4Cl tương ứng với các phản ứng này.


3. Hòa tan 10,00 g hỗn hợp gồm Cu2S và CuS bằng 200,0 mL dung dịch MnO4-0,7500 M trong môi trường axit.


Sau khi đun sơi để đuổi hết khí SO2sinh ra, lượng MnO4-còn dư trong dung dịch phản ứng vừa hết với 175,0
mL dung dịch Fe2+<sub>1,000 M.</sub>


(a) Viết các phương trình phản ứng xảy ra (dạng phương trình ion thu gọn).
(b) Tính phần trăm khối lượng CuS trong hỗn hợp ban đầu.


ĐÁP ÁN ĐIỂM


1. Trong thực tế, NH4Cl được dùng để đánh sạch bề mặt kim loại trước khi hàn:
4CuO + 2NH4ClN2+ 3Cu + CuCl2+ 4H2O


ZnO + 2NH4ClZnCl2+ 2NH3+ H2O


2. (a) Phương trình phản ứng:


5Cu2S + 8MnO4-+ 44H+10Cu2++ 5SO2+ 8Mn2++ 22H2O (1)
5CuS + 6MnO4-+ 28H+5Cu2++ 5SO2+ 6Mn2++ 14H2O (2)
5Fe2+<sub>+ MnO</sub>


4-+ 8H+5Fe3++ Mn2++ 4H2O (3)
(b) Xác định %


(1) 0,175 1 0,035mol


5
1
n
5
1
n 2



4(3) Fe


MnO      


<b>1,50</b>
(0,503)


</div>
<span class='text_page_counter'>(13)</span><div class='page_container' data-page=13>

 n<sub>MnO</sub> <sub>(</sub><sub>1</sub><sub>,</sub><sub>2</sub><sub>)</sub> 0,2 0,75 0,035 0,115mol
4









Đặt số mol Cu2S và CuS lần lượt là x và y, ta có:






















0625
,
0
y


025
,
0
x
115
,
0
y
5
6
x
5
8


10
y


96
x
160


 100% 60%


10
96
0625
,
0
m


% <sub>Cu</sub><sub>S</sub>    


<b>0,75</b>


<b>CÂU III</b>(4 điểm)


1. Chỉ dùng chất chỉ thị phenolphtalein, hãy phân biệt các dung dịch NaHSO4, Na2CO3, AlCl3, Fe(NO3)3, NaCl,
Ca(NO3)2. Các phản ứng minh họa viết dưới dạng ion thu gọn.


2. Hòa tan hết 2,2 g hỗn hợp kim loại A gồm sắt và nhôm trong 150 mL dung dịch HNO32M thu được dung dịch
B và 448 mL (đktc) khí C gồm N2O và N2có tỉ khối so với khơng khí bằng 1,2414. Thêm 13,6 g NaOH ngun
chất vào dung dịch B thu được kết tủa D, lọc kết tủa D thu được dung dịch nước lọc E.


(a) Tính phần trăm khối lượng mỗi kim loại trong hỗn hợp A ban đầu.


(b) Nung kết tủa D đến khối lượng khơng đổi thì thu được bao nhiêu gam chất rắn khan?
(c) Tính thể tích dung dịch HCl 0,5M cần thêm vào dung dịch E để thu được 2,34 g kết tủa.



<b>ĐÁP ÁN</b> <b>ĐIỂM</b>


1. Trích mẫu thử cho mỗi lần thí nghiệm:


 Cho phenolphtalein vào mỗi mẫu thử. Mẫu thử có màu hồng là dung dịch Na2CO3, các mẫu
thử còn lại không màu.


CO32-+ H2OHCO3-+ OH


- Dùng Na2CO3làm thuốc thử để cho vào các mẫu thử cịn lại.
Mẫu thử có sủi bọt khí khơng màu là NaHSO4


CO32-+ 2H+H2O + CO2↑


Mẫu thử tạo kết tủa trắng keo và sủi bọt khí khơng màu là AlCl3
2Al3+<sub>+ 3CO</sub>


32-+ 3H2O2Al(OH)3↓+ 3CO2↑


Mẫu thử tạo kết tủa đỏ nâu và sủi bọt khí khơng màu là Fe(NO3)3
2Fe3+<sub>+ 3CO</sub>


32-+ 3H2O2Fe(OH)3↓+ 3CO2↑
Mẫu thử tạo kết tủa trắng là Ca(NO3)2


Ca2+<sub>+ CO</sub>


32-CaCO3↓
Mẫu thử không tạo hiện tượng là NaCl.



2. (a) Đặt số mol N2O và N2lần lượt bằng a và b, ta có:


</div>
<span class='text_page_counter'>(14)</span><div class='page_container' data-page=14>

















01
,
0
b
a
36
29
2414
,
1
02


,
0
b
28
a
44
02
,
0
4
,
22
448
,
0
b
a


Đặt số mol Fe và Al lần lượt bằng x và y


Chất khử Chất oxi hóa


Fe - 3eFe3+ <sub>10H</sub>+<sub>+ 2NO</sub>


3-+ 8eN2O + 5H2O


3x x 0,10 0,08


Al - 3eAl3+ <sub>12H</sub>+<sub>+ 2NO</sub>



3-+ 10eN2+ 6H2O


3y y 0,12 0,10


Vì n (pu) 0,22mol n (bd) 0,3mol
H


H     nên axit dư, phản ứng khơng tạo Fe
2+<sub>.</sub>
Ta có:













04
,
0
y
02
,
0


x
18
,
0
y
3
x
3
2
,
2
y
27
x
56


Vậy 100% 50,9%


2
,
2
56
02
,
0
m


% <sub>Fe</sub>     và

%

m

<sub>Al</sub>

49

,

1

%



(b) Thêm NaOH vào dung dịch B [H+<sub>(0,15.2-0,22 = 0,08 mol), Fe</sub>3+<sub>(x = 0,02 mol), Al</sub>3+<sub>(y = 0,04</sub>


mol) và NO3-]


H+<sub>+ OH</sub>-<sub></sub><sub>H</sub>


2O (1)
Fe3+<sub>+ 3OH</sub>-<sub></sub><sub>Fe(OH)</sub>


3 (2)
Al3+<sub>+ 3OH</sub>-<sub></sub><sub>Al(OH)</sub>


3 (3)
Al(OH)3+ OH-AlO2-+ 2H2O (4)


mol
34
,
0
40
6
,
13
n
mol
3
,
0
n
4
n
3


n


n<sub>OH</sub><sub>(</sub><sub>1</sub><sub>,</sub><sub>2</sub><sub>,</sub><sub>3</sub><sub>,</sub><sub>4</sub><sub>)</sub>  <sub>H</sub>  <sub>Fe</sub>3  <sub>Al</sub>3   <sub>OH</sub><sub>(</sub><sub>bd</sub><sub>)</sub>  
sau (1), (2), (3), (4) vẫn còn dư OH-<sub>, kết tủa D là Fe(OH)</sub>


3(0,02mol)
2Fe(OH)3Fe2O3+ 3H2O,mrắn= mFe2O<sub>3</sub> 0,011601,6g
(c) Thêm HCl vào dung dịch E [Na+<sub>, OH</sub>-<sub>(0,04 mol), AlO</sub>


2-(0,04 mol) và NO3-]
OH-<sub>+ H</sub>+<sub></sub><sub>H</sub>


2O (5)
AlO2-+ H++ H2OAl(OH)3 (6)
Al(OH)3+ 3H+Al3++ 3H2O (7)


<b>1,00</b>


</div>
<span class='text_page_counter'>(15)</span><div class='page_container' data-page=15>

mol
03
,
0
78


34
,
2
n


3


)
OH
(


Al  


Trường hợp 1: Xảy ra (5), (6) và AlO2-dư


mol
07
,
0
03
,
0
04
,
0
n


n
n


3
)
OH
(
Al
OH



H       , <sub>0</sub><sub>,</sub><sub>5</sub> 0,14L
07


,
0


V 


Trường hợp 2: Xảy ra (5), (6), (7)
mol
01
,
0
03
,
0
04
,
0
n<sub>Al</sub><sub>(</sub><sub>OH</sub><sub>)</sub><sub>(</sub><sub>7</sub><sub>)</sub>


3   


mol
11
,
0
03
,
0


04
,
0
04
,
0
n


3
n


n


n<sub>H</sub> <sub>OH</sub> <sub>AlO</sub> <sub>Al</sub><sub>(</sub><sub>OH</sub><sub>)</sub> <sub>(</sub><sub>7</sub><sub>)</sub>
3


2     




  


  0,22L


5
,
0


11
,


0


V 


<b>1,00</b>
(0,502)


<b>CÂU IV</b>(3 điểm)


1. Tính hiệu ứng nhiệt phản ứng hidro hóa etilen tạo etan, biết nhiệt cháy của C2H6và C2H4lần lượt bằng -368,4
kcal/mol và -337,2 kcal/mol [sản phẩm cháy là CO2(k) và H2O (l)], nhiệt hình thành H2O (l) là -68,32 kcal/mol.


2. (a) Viết cơng thức cấu tạo và gọi tên anken ít cacbon nhất đồng thời có đồng phân hình học và đồng phân
quang học. (b) Viết các đồng phân hình học và quang học ứng với cấu tạo đó (sử dụng cơng thức Fisher) và xác
định cấu hình mỗi đồng phân (Z/E và R/S). (b) Viết cấu tạo các sản phẩm chính hình thành khi cho anken trên
tác dụng với dung dịch nước brom có lượng nhỏ muối natri clorua.


ĐÁP ÁN ĐIỂM


1. (1) C2H6(k) + 7/2O2(k)2CO2(k) + 3H2O (l) H368,4kcal
(2) C2H4(k) + 3O2(k)2CO2(k) + 2H2O (l) H337,2kcal
(3) H2(k) + 1/2O2(k)H2O (l) H68,32kcal
Lấy (2) - (1) + (3) ta được:


C2H4(k) + H2(k)C2H6(k) H(337,2)(368,4)(68,32)37,1kcal


2. (a) Cấu tạo:


CH<sub>3</sub> CH CH C



H


CH<sub>3</sub>


C<sub>2</sub>H<sub>5</sub> (4-metylhex-2-en)


(b) Cấu hình:


<b>1,00</b>


</div>
<span class='text_page_counter'>(16)</span><div class='page_container' data-page=16>

C
C


CH<sub>3</sub>
H


C<sub>2</sub>H<sub>5</sub>
C
H
H


CH<sub>3</sub>


C
C


H
CH<sub>3</sub>


C<sub>2</sub>H<sub>5</sub>



C
C


CH<sub>3</sub>
H


C<sub>2</sub>H<sub>5</sub>
C
CH<sub>3</sub>
H


H


C
C


H
CH<sub>3</sub>


C<sub>2</sub>H<sub>5</sub>
H


H
C
CH<sub>3</sub>


H
CH<sub>3</sub>
C


H


(E)(R) (E)(S)


(Z)(R) <sub>(Z)(S)</sub>


(c) Cấu tạo các sản phẩm:


CH<sub>3</sub> CH CH CH
CH<sub>3</sub>


C<sub>2</sub>H<sub>5</sub>


CH<sub>3</sub> CH CH CH
CH<sub>3</sub>


C<sub>2</sub>H<sub>5</sub>
Br Br


CH<sub>3</sub> CH CH CH
CH<sub>3</sub>


C<sub>2</sub>H<sub>5</sub>
OH Br


CH<sub>3</sub> CH CH CH
CH<sub>3</sub>


C<sub>2</sub>H<sub>5</sub>
Cl Br



<b>1,00</b>
(0,254)


<b>0,75</b>
(0,253)


<b>CÂU V (3 điểm)</b>


<b>1.</b> Hidrocacbon A có khối lượng phân tử bằng 80. Ozon phân A chỉ tạo andehit fomic và andehit oxalic.
C


H
O


H H C


O
C
O


H
andehit fomic andehit oxalic
<b>a.</b> Xác định cấu tạo và gọi tên A.


<b>b.</b> Dùng cơ chế giải thích các sản phẩm hình thành khi cộng Br2vào A theo tỉ lệ mol 1:1, gọi tên các sản phẩm
này.


<b>2.</b> Hợp chất A có cơng thức phân tử C9H8. A làm mất màu Br2trong CCl4; hidro hóa A trong điều kiện êm dịu tạo
ra C9H10,cịn trong điều kiện nhiệt độ và áp suất cao thì tạo ra C9H16; oxi hóa mãnh liệt A sinh ra axit phtalic


[1,2-C6H4(COOH)2]. Lập luận xác định cấu tạo của A.


ĐÁP ÁN ĐIỂM


1. (a) Công thức tổng quát cho A là CxHy
Ta có



















8
y


6
x
2


x
2
y


80
y
x
12


, cơng thức phân tử C6H8 (3)


</div>
<span class='text_page_counter'>(17)</span><div class='page_container' data-page=17>

CH<sub>2</sub> CH CH CH CH CH<sub>2</sub>


H<sub>2</sub>C O O HC HC O O HC HC O O CH<sub>2</sub>


A (hexa-1,3,5-trien)
(b) Cơ chế và sản phẩm:


CH<sub>2</sub> CH CH CH CH CH<sub>2</sub> Br2
CH<sub>2</sub> CH CH CH CH CH<sub>2</sub>


CH<sub>2</sub> CH CH CH CH CH<sub>2</sub>
Br


Br


CH<sub>2</sub> CH CH CH CH CH<sub>2</sub>
Br


CH<sub>2</sub> CH CH CH CH CH<sub>2</sub>



CH<sub>2</sub> CH CH CH CH CH<sub>2</sub>
Br


Br


CH<sub>2</sub> CH CH CH CH CH<sub>2</sub>
Br


Br


Br


Br
(X) 5,6-dibromhexa-1,3-dien; (Y) 3,6-dibromhexa-1,4-dien;


(X)


(Y)


(Z)


(Z) 1,6-dibromhexa-2,4-dien
2. A (C9H8) có độ bất bão hịa  6


A làm mất màu Br2và cộng êm dịu 1 phân tử H2cho thấy A có 1 liên kết đơi.


A cộng tối đa 4 phân tử H2và khi oxi hóa tạo axit phtalic cho thấy A có vịng benzen và ngồi ra
cịn một vịng 5 cạnh nữa.



Cơng thức của A:


<b>0,50</b>
(0,252)


<b>1,50</b>
(0,503)


<b>0,50</b>


<b>CÂU VI</b>(4 điểm)


Chia 3,584 L (đktc) hỗn hợp gồm một ankan (A), một anken (B) và một ankin (C) thành 2 phần bằng nhau. Phần 1
cho qua dung dịch AgNO3dư trong NH3thấy thể tích hỗn hợp giảm 12,5% và thu được 1,47g kết tủa. Phần 2 cho
qua dung dịch brom dư thấy khối lượng bình brom tăng 2,22g và có 13,6g brom đã tham gia phản ứng. Đốt cháy
hồn tồn khí ra khỏi bình brom rồi hấp thụ sản phẩm cháy vào dung dịch Ba(OH)2dư thì thu được 2,955g kết tủa.


1. Xác định công thức cấu tạo A, B và C.


2. Từ A viết dãy chuyển hóa (ghi rõ điều kiện phản ứng) điều chế 1,1-dibrompropan và 2,2-dibrompropan.
3. Viết phương trình phản ứng xảy ra khi cho C tác dụng với dung dịch KMnO4trong (i) mơi trường trung tính ở


</div>
<span class='text_page_counter'>(18)</span><div class='page_container' data-page=18>

ĐÁP ÁN ĐIỂM


1. Trong một phần, ta có: 0,08mol
4
,
22
584
,


3
2
1


n<sub>A</sub><sub>,</sub><sub>B</sub><sub>,</sub><sub>C</sub>   . Dung dịch AgNO3/NH3chỉ hấp thụ ankin,
đặt công thức ankin là RC≡CH (giả sử không phải là C2H2).


RC≡CH + AgNO3+ NH3RC≡CAg + NH4NO3 (1)
mol
01
,
0
08
,
0
100
5
,
12
n


n <sub>ankin</sub>    (R + 132)0,01 = 1,47
 R = 15 (CH3-), công thức của ankin là CH3C≡CH


Dung dịch brom hấp thụ anken (CnH2n) và ankin


CnH2n+ Br2CnH2nBr2 (2)
C3H4+ 2Br2C3H4Br4 (3)


g


82
,
1
40
01
,
0
22
,
2
m
n
2
nH


C     , 0,01 2 0,065mol
160


6
,
13
n<sub>Br</sub><sub>(</sub><sub>2</sub><sub>)</sub>


2    


Từ
065
,
0
1


82
,
1
n
14


 n = 2, cơng thức của anken là CH2=CH2.


Khí ra khỏi bình brom là ankan (CmH2m+2), nC<sub>n</sub>H<sub>2</sub><sub>n</sub><sub></sub><sub>2</sub> 0,080,010,0650,005mol
CmH2m+2+ O2


2
1
n
3





  <sub></sub>


nCO2+ (n+1)H2O (4)
CO2+ Ba(OH)2BaCO3+ H2O (5)


015
,
0
197
955


,
2
n
n
3
2 BaCO


CO    Từ (4): n 3


015
,
0
n
005
,
0


1 <sub></sub> <sub></sub> <sub></sub>


, công thức ankan là CH3CH2CH3.


2. Điều chế:


C<sub>3</sub>H<sub>8</sub> C<sub>3</sub>H<sub>7</sub>Cl CH<sub>3</sub>CH=CH<sub>2</sub> CH<sub>3</sub>CHBr-CH<sub>2</sub>Br


CH<sub>3</sub> C CH


CH<sub>3</sub>CH<sub>2</sub>CHBr<sub>2</sub>
CH<sub>3</sub>CBr<sub>2</sub>CH<sub>3</sub>



Cl2,as KOH/ROH


KOH/ROH


Br<sub>2</sub>
HBr
HBr
peoxit


3. Phản ứng của C:


+ 2KMnO<sub>4</sub>


CH<sub>3</sub> C CH CH<sub>3</sub> C C


O O


+ 2MnO<sub>2</sub>+ KOH
OK


5CH3C≡CH + 8KMnO4+ 12H2SO45CH3COOH + 5CO2


+ 8MnSO4+ 4K2SO4+ 12H2O


<b>0,75</b>


<b>0,75</b>


<b>0,50</b>



<b>1,00</b>
(0,502)


</div>
<span class='text_page_counter'>(19)</span><div class='page_container' data-page=19>

<b>ĐỀSỐ4</b>


<b>SỞ GIÁO DỤC VÀ ĐÀO TẠO</b>
<b>THÀNH PHỐ ĐÀ NẴNG</b>


<b>HƯỚNG DẪN CHẤM KÌ THI HỌC SINH GIỎI THÀNH PHỐ</b>
<b>NĂM HỌC 2005 - 2006</b>


<b>MƠN: HĨA HỌC LỚP 11</b>


Thời gian: 150 phút (không kể thời gian giao đề)


<b>CÂU I</b> (4 điểm)


1. Nêu hiện tượng xảy ra và viết phương trình phản ứng minh họa trong các trường hợp sau:


(a) Hòa tan từ từ cho đến dư dung dịch NaOH vào dung dịch AlCl3, sau đó thêm HCl vào dung dịch thu được
đến dư.


(b) Thêm dung dịch K2CO3vào dung dịch Fe(NO3)3


2. A là dung dịch Na2CO30,1M; B là dung dịch hỗn hợp Na2CO30,1M và KHCO30,1M và C là dung dịch KHCO3
0,1M.


(a) Tính thế tích khí CO2(đktc) thốt ra khi cho từ từ từng giọt đến hết 50 mL dung dịch HCl 0,1M vào 100 mL
dung dịch A và khi cho hết 100 mL dung dịch B vào 200 mL dung dịch HCl 0,1M.



(b) Xác định số mol các chất có trong dung dịch thu được khi thêm 100 mL dung dịch Ba(OH)20,1M vào 150
mL dung dịch C.


(c) Tính pH của các dung dịch A và C, biết axit cacbonic có pK1= 6,35 và pK2= 10,33.


(d) Đề nghị phương pháp nhận biết các anion có trong dung dịch B.


<b>ĐÁP ÁN</b> <b>ĐIỂM</b>


1. (a) Thêm dung dịch NaOH vào dung dịch AlCl3thấy xuất hiện kết tủa trắng keo, sau đó tan lại:
Al3+<sub>+ 3OH</sub>-<sub></sub><sub>Al(OH)</sub>


3


Al(OH)3+ OH-Al(OH)4


-Thêm HCl vào dung dịch thu được lại thấy xuất hiện kết tủa trắng keo, sau đó tan
lại:


Al(OH)4-+ H+Al(OH)3+ H2O
Al(OH)3+ 3H+Al3++ 3H2O


<b>0,50</b>


(b) Thêm dung dịch K2CO3vào dung dịch Fe(NO3)3thấy xuất hiện kết tủa đỏ nâu và sủi bọt khí
khơng màu: 2Fe3+<sub>+ 3CO</sub>


32-+ 3H2O2Fe(OH)3+ 3CO2


<b>0,25</b>



2. (a) Cho từ từ từng giọt đến hết 50 mL dung dịch HCl 0,1M vào 100 mL dung dịch Na2CO30,1M
CO32- + H+HCO3


-0,01 0,005


0,005 0,005


</div>
<span class='text_page_counter'>(20)</span><div class='page_container' data-page=20>

Do CO32-dư nên khơng có giai đoạn tạo CO2, VCO<sub>2</sub> 0


Cho hết 100 mL dung dịch Na2CO30,1M và KHCO30,1M vào 200 mL dung dịch HCl 0,1M:
CO32-+ 2H+H2O + CO2 (1)


HCO3-+ H+H2O + CO2 (2)
Vì 2n<sub>CO</sub>  n<sub>HCO</sub> n<sub>H</sub>


3
2


3 nên H


+<sub>phản ứng hết.</sub>


Giả sử (1) xảy ra trước thì ta có n 0,01mol
2


1
n


H



CO<sub>2</sub>   


Giả sử (2) xảy ra trước thì từ (1) và (2) ta có n 0,015mol
2


CO 


Thực tế (1) và (2) đồng thời xảy ra nên:


L
336
,
0
4
,
22
015
,
0
V


4
,
22
01
,
0
L
224


,
0


2


CO   






<b>1,00</b>


(b) Thêm 100 mL dung dịch Ba(OH)20,1M vào 150 mL dung dịch KHCO30,1M
HCO3- + OH-  CO32-+ H2O


0,015 0,02


0,015 0,015


0 0,005 0,015


Ba2+ <sub>+ CO</sub>


32-  BaCO3


0,01 0,015


0,01 0,01



0 0,005


Dung dịch còn 0,005 mol KOH và 0,005 mol K2CO3


<b>0,50</b>


(c) Dung dịch A có các cân bằng:


CO32-+ H2O⇌HCO3-+ OH- Kb1= 10-3,67
HCO3-+ H2O⇌H2O + CO2+ OH-Kb2= 10-7,65


H2O⇌H++ OH- KN= 10-14


Vì Kb1>> Kb2>> KNnên cân bằng (1) là chủ yếu:


</div>
<span class='text_page_counter'>(21)</span><div class='page_container' data-page=21>

pH = 14
-2
1


(pKb1+ pC) = 14
-2
1


(3,67 + 1) = 11,67
Dung dịch C là dung dịch lưỡng tính nên:


pH =
2
1



(pK1+ pK2) =
2
1


(6,35 + 10,33) = 8,34


(d) Trích mẫu thử, thêm BaCl2dư vào mẫu thử thấy xuất hiện kết tủa trắng (tan trong axit),
như vậy mẫu thử có CO32-.


Ba2+<sub>+ CO</sub>


32-BaCO3


Lọc tách kết tủa, thêm HCl vào dung dịch nước lọc thấy sủi bọt khí khơng màu (làm đục nước
vơi trong), vậy dung dịch có HCO3


-HCO3-+ H+H2O + CO2.


<b>0,50</b>


<b>CÂU II</b> (4 điểm)


1. (a) Amoniac có tính oxi hóa hay tính khử? Viết phương trình phản ứng minh họa. (b) Trong dung mơi amoniac


lỏng, các hợp chất KNH2, NH4Cl, Al(NH2)3có tính axit, bazơ hay lưỡng tính ? Viết các phương trình phản ứng
minh họa.


2. Hòa tan 4,8 gam kim loại M bằng dung dịch HNO3đặc nóng dư, hay hịa tan 2,4 gam muối sunfua kim loại này
cũng trong dung dịch HNO3đặc nóng, thì đều cùng sinh ra khí NO2duy nhất có thể tích bằng nhau trong cùng
điều kiện.



(a) Viết các phương trình phản ứng dưới dạng phương trình ion.


(b) Xác định kim loại M, cơng thức phân tử muối sunfua.


(c) Hấp thụ khí sinh ra ở cả hai phản ứng trên vào 300 mL dung dịch NaOH 1M, rồi thêm vào đó một ít
phenolphtalein. Hỏi dung dịch thu được có màu gì? Tại sao?


<b>ĐÁP ÁN</b> <b>ĐIỂM</b>


1. (a) NH3vừa có tính oxi hóa, vừa có tính khử :
Tính oxi hóa: K + NH3 (l)KNH2+ 1/2H2
Tính khử: 2NH3+ 3CuO3Cu + N2+ 3H2O


<b>0,75</b>


(b) KNH2là một bazơ, NH4Cl là axit và Al(NH2)3có tính lưỡng tính.
Phản ứng trung hịa: KNH2+ NH4ClKCl + 2NH3


Phản ứng của chất lưỡng tính với axit: Al(NH2)3+ 3NH4ClAlCl3+ 6NH3
Phản ứng của chất lưỡng tính với bazơ: Al(NH2)3+ KNH2K[Al(NH2)4]


<b>075</b>


2. (a) Phương trình phản ứng:
M + 2mH+<sub>+ mNO</sub>


</div>
<span class='text_page_counter'>(22)</span><div class='page_container' data-page=22>

(1)


M2Sn+ 4(m+n)H++ (2m+6n)NO3-2Mm++ nSO42-+ (2m+6n)NO2



+ 2(m+n)H2O (2)


<b>1,00</b>


(b) Vì số mol NO2ở hai trường hợp là bằng nhau nên ta có:
)


n
6
m
2
(
n
32
M
2


4
,
2
m


M
8
,


4 <sub></sub>












 


3
,
2
,
1
m
,
n


m
2
n
6


mn
64
M


, nghiệm thích hợp là n = 1, m = 2 và M = 64.



Vậy M là Cu và công thức muối là Cu2S.


<b>0,75</b>


(c) 0,075mol


64
8
,
4
n<sub>Cu</sub>  


Cu + 4HNO3Cu(NO3)2+ 2NO2+ 2H2O


 n<sub>NO</sub> 2 2 0,075 0,3mol n<sub>NaOH</sub>
2     
đã xảy ra vừa đủ phản ứng:


2NO2+ 2NaOHNaNO3+ NaNO2+ H2O


Dung dịch thu được có màu hồng do NO2-tạo môi trường bazơ:
NO2-+ H2O⇌HNO2+ OH


<b>-0,75</b>


<b>CÂU III</b>(4 điểm)


1. (a) Tính tỉ lệ các sản phẩm monoclo hóa (tại nhiệt độ phịng) và monobrom hóa (tại 127oC) isobutan. Biết tỉ lệ
khả năng phản ứng tương đối của nguyên tử H trên cacbon bậc nhất, bậc hai và bậc ba trong phản ứng clo


hóa là 1,0 : 3,8 : 5,0 và trong phản ứng brom hóa là 1 : 82 : 1600.


(b) Dựa vào kết quả tính được ở câu (a), cho nhận xét về các yếu tố ảnh hưởng đến hàm lượng các sản phẩm
của phản ứng halogen hóa ankan.


2. Dùng cơ chế phản ứng giải thích tại sao khi xử lý 2,7-đimetylocta-2,6-dien với axit photphoric thì thu được
1,1-đimetyl-2-isopropenylxiclopentan.


3. Hiđro hóa một hiđrocacbon A (C8H12) hoạt động quang học thu được hiđrocacbon B (C8H18) không hoạt động
quang học. A không tác dụng với Ag(NH3)2+và khi tác dụng với H2trong sự có mặt của Pd/PbCO3tạo hợp chất
khơng hoạt động quang học C (C8H14).


4. Lập luận xác định cấu tạo (có lưu ý cấu hình) và gọi tên A, B, C.


</div>
<span class='text_page_counter'>(23)</span><div class='page_container' data-page=23>

<b>ĐÁP ÁN</b> <b>ĐIỂM</b>


1. (a) Tỉ lệ sản phẩm:


CH<sub>3</sub> CH CH<sub>3</sub>


+ Cl2


- HCl


CH<sub>3</sub> CH CH<sub>2</sub> Cl


CH<sub>3</sub> C CH<sub>3</sub>
Cl


1-clo-2-metylpropan



2-clo-2-metylpropan
CH<sub>3</sub>


CH<sub>3</sub>


CH<sub>3</sub>


(9x1,0)


(9x1,0) + (1x5,0)<b>= 64,3%</b>


(1x5,0)


(9x1,0) + (1x5,0)<b>= 35,7%</b>


CH3 CH CH3


+ Br2


- HCl


CH<sub>3</sub> CH CH<sub>2</sub> Br


CH<sub>3</sub> C CH<sub>3</sub>
Br


1-brom-2-metylpropan


2-brom-2-metylpropan


CH3


CH<sub>3</sub>


CH<sub>3</sub>


(9x1,0)


(9x1,0) + (1x1600) <b>= 0,56%</b>


(1x1600)


(9x1,0) + (1x1600)<b>= 99,44%</b>


<b>0,50</b>


<b>0,50</b>


(b) Hàm lượng sản phẩm halogen hóa phụ thuộc ba yếu tố:


 Khả năng tham gia phản ứng thế của ankan: Phản ứng halogen hóa ưu tiên thế hidro trên
nguyên tử cacbon bậc cao hơn.


 Khả năng phản ứng của halogen: Brom tham gia phản ứng yếu hơn so với clo, nhưng có
khả năng chọn lọc vị trí thế cao hơn so với clo.


 Số nguyên tử hidro trên cacbon cùng bậc: Khi số hidro trên các nguyên tử cacbon càng
nhiều thì hàm lượng sản phẩm càng lớn.


<b>0,75</b>



2. Cơ chế:


H+


-H+


<b>0,75</b>


3. (a) A có độ bất bão hịa 3
2


12
8
.
2


2  <sub></sub>




 , B có 0


2
18
8
.
2


2  <sub></sub>





 và C có


2
2


14
8
.
2


2  <sub></sub>




</div>
<span class='text_page_counter'>(24)</span><div class='page_container' data-page=24>

 Vì A cộng 3 phân tử hidro để tạo ra B nên A có các liên kết bội hoặc vịng ba cạnh.


 A cộng 1 phân tử H2tạo ra C và A không tác dụng với Ag(NH3)2+nên A có một liên kết ba
dạng -CC-R.


 A cũng phải chứa một liên kết đôi dạng<i>cis-</i>(Z) ở vị trí đối xứng với liên kết ba, vì khi A cộng
1 phân tử H2 (xúc tác Pd làm cho phản ứng chạy theo kiểu<i>cis-</i>) tạo C không hoạt động
quang học.


<b>0,50</b>
Cấu tạo của A, B, C là:


CH3 C C



H H


H
C
CH<sub>3</sub>


C


* <sub>C</sub> <sub>CH</sub>


3 2Z-4-metylhept-2-en-5-in


<b>(A)</b>


CH3CH2CH2CH(CH3)CH2CH2CH3 4-metylheptan
<b>(B)</b>


CH<sub>3</sub> C C
H H


H
C
CH<sub>3</sub>


C C CH<sub>3</sub> 2Z,5Z-4-metylhepta-2,5-dien
<b>(C)</b>


H H



<b>0,75</b>


(b) Phương trình phản ứng:


<b>5CH</b>3CH=CHCH(CH3)CC-CH3+<b>14KMnO</b>4+<b>21H</b>2SO4


<b>10CH</b>3COOH +<b>5CH</b>3CH(COOH)2+<b>14MnSO</b>4+<b>7K</b>2SO4+<b>16H</b>2O


<b>0,25</b>


<b>CÂU IV(4 điểm)</b>


1. <i>Limonen</i>(C10H16) là tecpen có trong vỏ quả cam, chanh và bưởi. Oxi hóa limonen bằng kalipemanganat tạo
chất A.


CH<sub>3</sub> C
O


CH<sub>2</sub> CH<sub>2</sub> CH


CH<sub>2</sub>COOH


C O


H3C


(A)


(a) Dùng dữ kiện trên và qui tắc isopren xác định cấu trúc của limonen.
(b) Viết công thức các sản phẩm chính hình thành khi hidrat hóa limonen.



2. Để điều chế nitrobenzen trong phịng thí nghiệm và tính hiệu suất phản ứng, người ta tiến hành các bước sau:


Cho 19,5 ml axit nitric vào một bình cầu đáy trịn cỡ 200 mL làm lạnh bình và lắc, sau đó thêm từ từ 15 mL
H2SO4đậm đặc, đồng thời lắc và làm lạnh đến nhiệt độ phòng. Lắp ống sinh hàn hồi lưu (nước hay khơng khí),
cho tiếp 13,5 mL benzen qua ống sinh hàn với tốc độ chậm và giữ nhiệt độ không quá 500<sub>C, đồng thời lắc liên</sub>
tục (a).


</div>
<span class='text_page_counter'>(25)</span><div class='page_container' data-page=25>

(a) Viết phương trình hố học chính và các phương trình thể hiện cơ chế của phản ứng. Cho biết vì sao cần
phải lắc bình liên tục và giữ nhiệt độ phản ứng ở 500<sub>C? Nếu không dùng H</sub>


2SO4đậm đặc, phản ứng có xảy
ra khơng?


(b) Vì sao cần phải rửa nitrobenzen bằng nước, sau đó bằng dung dịch Na2CO3?
(c) A có thể là chất nào?


(d) Tính hiệu suất phản ứng nếu khối lượng riêng của benzen 0,8g/mL.


<b>ĐÁP ÁN</b> <b>ĐIỂM</b>


1. (a) Cấu tạo:


O + O=C=O
O


O
OH


limonen



<b>0,75</b>
(b) Các sản phẩm chính khi hidrat hóa:


OH


OH OH


OH


<b>0,75</b>


4. (a) Phản ứng:


C6H6+ HONO2 H 2SO4 C6H5NO2+ H2O (1)
Cơ chế phản ứng:


HO - NO<sub>2</sub>+ H<sub>2</sub>SO<sub>4</sub> H - O - NO<sub>2</sub> + HSO<sub>4</sub>
H


(+) <sub>(-)</sub>


H - O - NO<sub>2</sub> + H<sub>2</sub>SO<sub>4</sub>
H


(+)


H<sub>3</sub>O(+)+ HSO<sub>4</sub>(-) + NO<sub>2</sub>
(+)



</div>
<span class='text_page_counter'>(26)</span><div class='page_container' data-page=26>

+ NO<sub>2</sub>(+)


H NO2


+


NO<sub>2</sub>


+ H(+)


chËm nhanh


 Hỗn hợp phản ứng ở hệ dị thể nên cần phải lắc đều hay khuấy mạnh liên tục để tạo thành
nhũ tương, bảo đảm sự tiếp xúc tốt giữa các tác nhân.


 Phải giữ ở 500<sub>C vì nếu ở nhiệt độ cao hơn sẽ tăng lượng sản phẩm đinitrobenzen.</sub>


 Nếu không dùng H2SO4, phản ứng vẫn xảy ra do vẫn có sự hình thành NO2+theo phương
trình sau:


HO-NO2+ HNO3⇄H2O+-NO2+ NO3
-H2O+-NO2+ HNO3⇄H3O++ NO3-+ NO2+(1)


Tuy nhiên khi khơng có H2SO4phản ứng xảy ra chậm vì hiệu suất tạo NO2+sinh ra trong (1)
rất thấp. Khi có mặt H2SO4đậm đặc, cân bằng chuyển dời về phía thuận nên phản ứng xảy
ra nhanh hơn.


<b>0,75</b>


(c) Cần phải rửa bằng nước để loại axit, sau đó rửa bằng dung dịch Na2CO3để loại hết axit dư


và dễ kiểm tra kết quả do phản ứng giữa axit và Na2CO3sinh khí.


<b>0,25</b>


(b) A là chất hút nước ở dạng rắn, nên A có thể là CaCl2, ... khan <b>0,25</b>
(d) Hiệu suất phản ứng:


gam
512
,
9
123


78
15
m<sub>C</sub> <sub>H</sub> <sub>(</sub><sub>1</sub><sub>)</sub>


6


6 




  88%


mL
/
g
8
,


0
mL
5
,
13


g
512
,
9


H 




 <b>0,50</b>


<b>CÂU VI</b>


A là hidrocacbon không làm mất màu dung dịch brom. Đốt cháy hoàn toàn 0,02 mol A và hấp thu sản phẩm cháy
vào dung dịch chứa 0,15 mol Ca(OH)2thu được kết tủa và khối lượng bình tăng lên 11,32 gam. Cho dung dịch
Ba(OH)2dư vào dung dịch thu được kết tủa lại tăng lên, tổng khối lượng kết tuả hai lần là 24,85 gam. A khơng với
dung dịch KMnO4/H2SO4nóng, cịn khi monoclo hóa trong điều kiện chiếu sáng thì chỉ tạo một sản phẩm duy
nhất.


1. Xác định cơng thức cấu tạo và gọi tên A.


2. Người ta có thể điều chế A từ phản ứng giữa benzen và anken tương ứng trong axit sunfuric. Dùng cơ chế
phản ứng để giải thích phản ứng này.



3. Mononitro hóa A bằng cách cho phản ứng với axit nitric (có mặt axit sunfuric đặc) thì sản phẩm chính thu
được là gì? Tại sao?


<b>ĐÁP ÁN</b> <b>ĐIỂM</b>


1. Dung dịch Ca(OH)2hấp thụ hết sản phẩm cháy của A chứa CO2và H2O


</div>
<span class='text_page_counter'>(27)</span><div class='page_container' data-page=27>

2CO2+ Ca(OH)2Ca(HCO3)2 (2)


Ca(HCO3)2+ Ba(OH)2CaCO3+ BaCO3+ 2H2O (3) <b>0,50</b>


Đặt số mol CO2tham gia các phản ứng (1) và (2) lần lượt là x và y, ta có:


mol
1
,
0
y
x
85
,
24
2
y
197
2
y
x
100
15


,
0
2
y
x
















 



, n x y 0,2mol
2


CO   



Từ 0,14mol


18
44
.
2
,
0
32
,
11
n
g
32
,
11
m
m


m <sub>H</sub> <sub>O</sub> <sub>CO</sub> <sub>H</sub> <sub>O</sub>


2
2
2 







 <b>0,50</b>


Đặt công thức tổng quát của A là CxHy:
CxHy+ (x+y/4)O2xCO2+ y/2H2O


Ta có x 10, y 14


14
,
0
.
2
y
2
,
0
x
02
,
0


1 <sub></sub> <sub></sub> <sub></sub> <sub></sub> <sub></sub>


Cơng thức phân tử của A là C10H14

4



<b>0,50</b>


Vì A không làm mất màu dung dịch brom (cấu trúc thơm), không tác dụng với dung dịch
KMnO4/H2SO4 (chỉ có một nhóm thế) và monoclo hóa (ánh sáng) chỉ tạo một sản phẩm duy
nhất (nhóm thế có cấu trúc đối xứng cao) nên cấu tạo của A là:



C
CH<sub>3</sub>
CH3
CH<sub>3</sub>
<i>(t-butylbenzen)</i>
<b>1,00</b>


2. Cơ chế:


(CH3)2C=CH2+ H2SO4(CH3)2C+-CH3+ HSO4


-+


C(CH<sub>3</sub>)<sub>3</sub>
H


+ H(+)


chËm nhanh


+ (CH<sub>3</sub>)<sub>3</sub>C+


C(CH<sub>3</sub>)<sub>3</sub>


<b>1,00</b>


3. Nhóm ankyl nói chung định hướng thế vào các vị trí<i>ortho-</i>và<i>para-</i>. Tuy nhiên, do nhóm<i>t-</i>butyl


có kích thước lớn gây án ngữ khơng gian nên sản phẩm chính là sản phẩm<i>para-</i>:


C


CH3


CH<sub>3</sub>
CH<sub>3</sub>
O<sub>2</sub>N


</div>
<span class='text_page_counter'>(28)</span><div class='page_container' data-page=28>

ĐỀ SỐ 5


<b>SỞ GIÁO DỤC VÀ ĐÀO TẠO</b>
<b>THÀNH PHỐ ĐÀ NẴNG</b>


<b>ĐỀ CHÍNH THỨC</b>
Đề này có hai (2) trang


<b>HƯỚNG DẪN CHẤM ĐỀ THI HỌC SINH GIỎI THÀNH PHỐ</b>
<b>NĂM HỌC 2006 - 2007</b>


<b>MƠN: HĨA HỌC LỚP 11</b>


Thời gian: 150 phút (khơng kể thời gian giao đề)


<b>CÂU I</b> (4 điểm)


1. Tính pH của dung dịch thu được khi trộn lẫn 50,0 mL dung dịch NH4Cl 0,200 M với 75,0 mL dung dịch NaOH
0,100 M. Biết Kb(NH3) = 1,8.10-5.


2. Phèn là muối sunfat kép của một cation hóa trị một (như K+hay NH4+) và một cation hóa trị ba (như Al3+, Fe3+
hay Cr3+<sub>). Phèn sắt amoni có cơng thức (NH</sub>



4)aFe(SO4)b.nH2O. Hịa tan 1,00 gam mẫu phèn sắt vào 100 cm3
H2O, rồi chia dung dịch thu được thành hai phần bằng nhau. Thêm dung dịch NaOH dư vào phần một và đun
sôi dung dịch. Lượng NH3thoát ra phản ứng vừa đủ với 10,37 cm3dung dịch HCl 0,100 M. Dùng kẽm kim loại
khử hết Fe3+<sub>ở phần hai thành Fe</sub>2+<sub>. Để oxi hóa ion Fe</sub>2+<sub>thành ion Fe</sub>3+<sub>trở lại, cần 20,74 cm</sub>3<sub>dung dịch KMnO</sub>


4
0,0100 M trong mơi trường axit.


(a) Viết các phương trình phản ứng dạng ion thu gọn và xác định các giá trị a, b, n.
(b) Tại sao các phèn khi tan trong nước đều tạo môi trường axit ?


<b>ĐÁP ÁN</b> <b>ĐIỂM</b>


1. 0,08M


L
125
,
0
L
.
mol
200
,
0
L
050
,
0


C
1
o
Cl
NH4 


  ; 0,06M


L
125
,
0
L
.
mol
100
,
0
L
075
,
0
C
1
o
NaOH 

 



NH4Cl + NaOHNaCl + NH3+ H2O
0,08 0,06


0,06 0,06 0,06
0,02 0 0,06
Xét cân bằng :


NH3+ H2O⇄NH4++ OH
-0,06 0,02


x x x
0,06–x 0,02+x x


5
3


4


b 1,8.10


x
06
,
0
x
)
x
02
,
0


(
]
NH
[
]
OH
][
NH
[
K 







 , gần đúng 5,4.10 M


02
,
0
06
,
0
10
.
8
,
1



x 5  5


 pH14[lg(5,4.105)]9,73


</div>
<span class='text_page_counter'>(29)</span><div class='page_container' data-page=29>

<b>1,00</b>
2. (a) Đặt số mol của phèn sắt (NH4)aFe(SO4)b.nH2O trong mỗi phần là x mol.


Phương trình phản ứng phần một :
NH4++ OH-NH3+ H2O


ax


0 ax


Fe3+<sub>+ 3OH</sub>-<sub></sub><sub>Fe(OH)</sub>
3


NH3+ H+NH4+
ax ax


Phương trình phản ứng phần hai :
Zn + 2Fe3+<sub></sub><sub>Zn</sub>2+<sub>+ 2Fe</sub>2+


x


0 x
5Fe2+<sub>+ MnO</sub>


4-+ 8H+5Fe3++ Mn2++ 4H2O


x x/5


<b>10,25</b>


Ta có : ax0,01037L0,100mol.L11,037.103mol
mol
10
.
037
,
1
L
.
mol
010
,
0
L
02074
,
0
5


x   1 3


 a = 1


Công thức của phèn được viết lại là NH4+Fe3+(SO42-)b.nH2O


 b = 2



Từ M = 18 + 56 + 96.2 + 18n =


mol
10
.
037
,
1


gam
5
,
0


3

 n = 12


Công thức của phèn sắt – amoni là NH4Fe(SO4)2.12H2O


(b) Phèn tan trong nước tạo mơi trường axit vì các ion NH4+, Al3+, Fe3+và Cr3+đều những ion
axit (các ion K+<sub>có tính trung tính, cịn SO</sub>


42-có tính bazơ rất yếu).
NH4++ H2O⇄NH3+ H3O+


M3+<sub>+ H</sub>


</div>
<span class='text_page_counter'>(30)</span><div class='page_container' data-page=30>

<b>CÂU II (4 điểm)</b>



1. Viết phương trình phản ứng xảy ra khi lần lượt cho các đơn chất As và Bi tác dụng với dung dịch HNO3(giả
thiết sản phẩm khử chỉ là khí NO).


2. So sánh (có giải thích) tính tan trong nước, tính bazơ và tính khử của hai hợp chất với hidro là amoniac (NH3)
và photphin (PH3).


3. Một giai đoạn quan trọng trong quá trình tổng hợp axit nitric là oxi hóa NH3trong khơng khí, có mặt Pt xúc
tác.


(a) Xác định nhiệt phản ứng của phản ứng này, biết nhiệt hình thành các chất NH3(k), NO (k) và H2O (k) lần
lượt bằng – 46 kJ/mol; + 90 kJ/mol và - 242 kJ/mol.


(b) Trong công nghiệp, người ta đã sử dụng nhiệt độ và áp suất thế nào để quá trình này là tối ưu ? Tại sao ?


<b>ĐÁP ÁN</b> <b>ĐIỂM</b>


1. Phương trình phản ứng :


3As + 5HNO3+ 2H2O3H3AsO4+ 5NO
Bi + 4HNO3Bi(NO3)3+ NO + 2H2O


1,00


2. Tính tan :


NH3tan tốt hơn PH3trong nước, do phân tử phân cực hơn và có khả năng tạo liên kết hidro
với nước.


H N


H


H


H O
H


H N
H
H


<b>...</b> <b>...</b> <b>...</b>


<b>...</b>


Tính bazơ :


NH3có tính bazơ mạnh hơn PH3, do liên kết N-H phân cực mạnh hơn liên kết P-H, làm cho
nguyên tử N trong phân tử NH3giàu electron hơn, dễ dàng nhận proton hơn (một nguyên
nhân nữa giải thích cho điều này là ion NH4+bền hơn PH4+).


Tính khử :


PH3có tính khử mạnh hơn nhiều so với NH3, do nguyên tử P là một phi kim có độ âm điện
nhỏ và phân tử PH3kém bền hơn NH3.


3. (a) 4NH3(k) + 5O2(k)4NO (k) + 6H2O (k)












3


2O NH


H


NO 6 H 4 H


H
4
H


kJ
908
)
kJ
46
(
4
[
)]
kJ
242


(
6
[
)
kJ
90
4
(


H        




</div>
<span class='text_page_counter'>(31)</span><div class='page_container' data-page=31>

<b>CÂU III</b>(4 điểm)


1. Dùng hình vẽ, mơ tả thí nghiệm được tiến hành trong phịng thí nghiệm để xác định sự có mặt của các ngun
tố C và H có trong glucozơ.


2. Hồn thành các phản ứng dưới đây. Xác định sản phẩm chính của mỗi phản ứng và dùng cơ chế giải thích sự
hình thành sản phẩm chính đó.


(a) CH3-CH=CH2(propilen) + HCl


(b) CH3-CH2-CH(OH)-CH3(ancol<i>s-</i>butylic) HSO,180C
o
4
2


(c) C6H5CH3+ HNO3 
o


4
2SO ,t
H


3. Dùng sơ đồ xen phủ obitan nguyên tử để mô tả các phân tử CH3-CH=C=CH-CH3 (phân tử A) và CH3
-CH=C=C=CH-CH3(phân tử B). Cho biết A, B có đồng phân hình học hay khơng ? Tại sao ?


<b>ĐÁP ÁN</b> <b>ĐIỂM</b>


1. Thí nghiệm xác định sự có mặt của các nguyên tố C và H có trong glucozơ :


<b>1,5</b>


2. Phản ứng và cơ chế phản ứng:
(a) Phản ứng :


CH3 CH CH2 + HCl


CH


CH3 CH3


Cl
CH<sub>2</sub>


CH<sub>3</sub> CH<sub>2</sub> Cl


(s¶n phÈm chÝnh)


Cơ chế (cộng AE) :



CH<sub>3</sub> CH CH<sub>2</sub> H


+


 CH3 CH CH3


CH3 CH2 CH2


Cl


-CH


CH3 CH3


Cl


<b>(X)</b>


<b>(Y)</b>


Sản phẩm chính hình thành theo hướng tạo cacbocation trung gian bền vững hơn. Dễ
thấy rằng cacbocation (X) bền hơn (Y) (do điện tích được giải tỏa nhiều hơn, với 6Hα<sub>),</sub>
nên sản phẩm chính là isopropyl clorua.


</div>
<span class='text_page_counter'>(32)</span><div class='page_container' data-page=32>

(b) Phản ứng :


CH3 CH2


H<sub>2</sub>SO<sub>4</sub>



(s¶n phÈm chÝnh)


CH CH3


OH


CH3 CH CH CH3+ H2O


CH2 CH CH2 CH3+ H2O
Cơ chế (tách E1) :


CH3 CH2 CH CH3


OH


CH3 CH CH CH3


CH2 CH CH2 CH3


H+


CH3 CH2 CH CH3
+


OH<sub>2</sub> -H2O


<b>(X)</b>


<b>(Y)</b>



Sản phẩm chính được hình thành theo hướng tạo sản phẩm bền hơn. Ở đây, (X) bền hơn


(Y) do có số nguyên tử Hα<sub>tham gia liên hợp, làm bền hóa liên kết π nhiều hơn.</sub> <b>0,50</b>


(c) Phản ứng :


+ HONO<sub>2</sub>


H2SO4


+ H<sub>2</sub>O
CH<sub>3</sub>


CH<sub>3</sub>


NO<sub>2</sub>


+ H<sub>2</sub>O
CH<sub>3</sub>


NO<sub>2</sub>


Cơ chế (thế SE2Ar) : HONO2+ H2SO4HSO4-+ H2O ++NO2


+<sub>NO</sub>
2


+<sub>NO</sub>
2



-H+
CH<sub>3</sub> CH<sub>3</sub>


CH<sub>3</sub> CH3


-H+


CH<sub>3</sub> CH<sub>3</sub>
H


NO<sub>2</sub>


H NO2


NO<sub>2</sub>


NO<sub>2</sub>


Phản ứng dịnh hướng thế vào vị trí<i>meta-</i>, do mật độ electron ở vị trí này trong phân tử
toluen giàu hơn các vị trí ortho-, para-. Đồng thời phản ứng thế vào vị trí này tạo sự giải
tỏa điện tích tốt nhất ở phức π.


<b>0,50</b>


3. Mơ hình phân tử :


H


CH3



</div>
<span class='text_page_counter'>(33)</span><div class='page_container' data-page=33>

Trong truờng hợp này, các nhóm thế khơng đồng phẳng, nên phân tử khơng xuất hiện hiện
tượng đồng phân hình học.


H
CH3


H
CH3


Trong trường hợp này, các nhóm thế đồng phẳng, nên phân tử xuất hiện hiện tượng đồng
phân hình học.


<b>1,00</b>


<b>CÂU IV</b>(4 điểm)


1. Thổi 672 mL (đktc) hỗn hợp khí A gồm một ankan, một anken và một ankin (đều có số nguyên tử cacbon trong
phân tử bằng nhau) qua dung dịch AgNO3/NH3, thì thấy có 3,4 AgNO3đã tham gia phản ứng. Cũng lượng hỗn
hợp khí A trên làm mất màu vừa hết 200 mL dung dịch Br20,15 M.


(a) Xác định thành phần định tính và định lượng các chất trong A
(b) Đề nghị phương pháp tách riêng từng chất ra khỏi hỗn hợp A.


2. Oxi hóa m gam hợp chất hữu cơ A bằng CuO rồi cho sản phẩm sinh ra gồm CO2và hơi H2O lần lượt đi qua bình
1 đựng Mg(ClO4)2và bình 2 đựng 2 lít Ca(OH)20,0 2 M thì thu được 2 gam kết tủa. Khối lượng bình 1 tăng
1,08 gam và khối lượng CuO giảm 3,2 gam, MA< 100. Oxi hóa mãnh liệt A, thu được hai hợp chất hữu cơ là
CH3COOH và CH3COCOOH.


(a) Xác định công thức cấu tạo và gọi tên A.



(b) Viết các dạng đồng phân hình học tương ứng của A.


(c) Khi cho A tác dụng với Br2theo tỉ lệ mol 1:1, thì tạo được những sản phẩm nào ? Giải thích.


<b>ĐÁP ÁN</b> <b>ĐIỂM</b>


1. (a) Nếu ankin có dạng RCCH :


RCCH + AgNO3+ NH3RCCAg + NH4NO3


 0,02mol


mol
/
gam
170


gam
4
,
3
)
ankin
(


n   và n 2 n(ankin) 0,04mol


2



Br   


Điều này trái giả thiết, vì số mol Br2chỉ bằng0,2L0,15mol/L0,03mol
Vậy ankin phải là C2H2và như vậy ankan là C2H6, anken là C2H4.


Từ phản ứng :


C2H2+ 2AgNO3+ 2NH3C2Ag2+ 2NH4NO3


 n(C2H2) = 1/2n(AgNO3) = 0,01 mol
Từ các phản ứng :


C2H2+ 2Br2C2H2Br4
C2H4+ Br2C2H4Br2


 n(C2H4) = 0,01 mol


</div>
<span class='text_page_counter'>(34)</span><div class='page_container' data-page=34>

 n(C2H6) = 0,01mol0,01mol


mol
/
L
4
,
22


L
672
,
0



0,01 mol


(b) Thổi hỗn hợp qua binh chứa dung dịch AgNO3/NH3dư. Lọc tách kết tủa, hòa tan kết tủa
trong dung dịch HCl dư thu được khí C2H2.


C2H2+ 2AgNO3+ 2NH3C2Ag2+ 2NH4NO3
C2Ag2+ 2HClC2H2+ 2AgCl


Khí ra khỏi bình chứa dung dịch AgNO3/NH3, thổi tiếp qua dung dịch nước brom dư. Chiết lấy
sản phẩm và đun nóng với bột Zn (trong CH3COOH) thu được C2H4:


C2H4+ Br2C2H4Br2
C2H4Br2+ ZnC2H4+ ZnBr2


Khí ra khỏi bình chứa dung dịch brom là khí C2H6


<b>1,00</b>


2. (a) n(H2O) = 0,06 moln(H) = 0,12 mol
Từ các phản ứng :


CO2+ Ca(OH)2CaCO3+ H2O
2CO2+ Ca(OH)2Ca(HCO3)2
với n 0,045mol


2
)
OH
(



Ca  và nCaCO3 0,02mol n(CO2) bằng 0,02 mol hoặc 0,07 mol.
n(O) tham gia phản ứng bằng 0,2mol


mol
/
gam
16


gam
2
,


3 <sub></sub>


Vậy số mol O trong A bằng :


n(O) = 0,02mol2 + 0,06 mol – 0,2 mol < 0 (loại)


n(O) = 0,07mol2 + 0,06 mol – 0,2 mol = 0 mol


A là hidrocacbon có cơng thức đơn giản C7H12


Vì MA< 100, nên cơng thức phân tử của A chính là C7H12(2)
Cấu tạo của A phù hợp với giả thiết là:


CH


CH<sub>3</sub> C



CH<sub>3</sub>


CH CH CH<sub>3</sub> (3-metylhexa-2,4-dien)


<b>1,00</b>


<b>0,50</b>


</div>
<span class='text_page_counter'>(35)</span><div class='page_container' data-page=35>

CH<sub>3</sub>
C C


CH<sub>3</sub>
C C


CH<sub>3</sub>
H
H
H


H<sub>3</sub>C
C C


CH<sub>3</sub>
C C


H
CH<sub>3</sub>
H


H



H
C C


CH<sub>3</sub>
C C


CH<sub>3</sub>
H
H
CH<sub>3</sub>


H
C C


CH<sub>3</sub>
C C


H
CH<sub>3</sub>
H


CH<sub>3</sub>


<i>cis-cis</i> <i>cis-trans</i> <i>trans-cis</i> <i>trans-trans</i>


<b>0,50</b>


(c) Tác dụng với brom theo tỉ lệ mol 1:1 thì tạo được các sản phẩm :



CH


CH<sub>3</sub> C


CH<sub>3</sub>


CH CH CH<sub>3</sub> + Br2
- Br


-CH


CH<sub>3</sub> C


CH<sub>3</sub>
H


C CHBr CH<sub>3</sub>
H


C


CH<sub>3</sub> C


CH<sub>3</sub>


CH CHBr
H


C



CH<sub>3</sub> C


CH<sub>3</sub>


CH CH CH<sub>3</sub>
CH<sub>3</sub>


Br


+ Br


-CH<sub>3</sub>CH=C(CH<sub>3</sub>)-CHBr-CHBr-CH<sub>3</sub>
CH<sub>3</sub>-CHBr-C(CH<sub>3</sub>)=CH-CHBr-CH<sub>3</sub>
CH<sub>3</sub>-CHBr-CBr(CH<sub>3</sub>)-CH=CH-CH<sub>3</sub>


<b>0,50</b>


<b>CÂU V( 4 điểm)</b>


1. Trình bày phương pháp phân biệt mỗi cặp chất dưới đây (mỗi trường hợp chỉ dùng một thuốc thử đơn giản,
có viết phản ứng minh họa) :


(a) <i>m-</i>bromtoluen và benzylbromua
(b) phenylaxetilen và styren


2. Từ benzen và các chất vô cơ, xúc tác cần thiết khác có đủ, viết các phương trình phản ứng hóa học điều chế :
(a) <i>meta-</i>clonitrobenzen


(b) <i>ortho</i>-clonitrobenzen
(c) axit<i>meta-</i>brombenzoic


(d) axit<i>ortho-</i>brombenzoic


3. Hidrocacbon X có phân tử khối bằng 128, khơng làm nhạt màu dung dịch Br2. X tác dụng với H2(xúc tác Ni, t)
tạo các sản phẩm Y và Z. Oxi hóa mãnh liệt Y tạo sản phẩm là axit<i>o-</i>phtalic,<i>o-</i>C6H4(COOH)2.


(a) Xác định cấu tạo và gọi tên X, Y, Z.


(b) Viết phản ứng tạo ra sản phẩm chính, khi cho X lần lượt tác dụng với dung dịch HNO3đặc (H2SO4đặc xúc
tác) và Br2(xúc tác bột sắt). Biết ở mỗi phản ứng, tỉ lệ mol các chất tham gia phản ứng là 1:1.


<b>ĐÁP ÁN</b> <b>ĐIỂM</b>


1. Phân biệt các chất :


</div>
<span class='text_page_counter'>(36)</span><div class='page_container' data-page=36>

(b) Dùng dung dịch AgNO3/NH3, phenylaxetilen cho kết tủa vàng xám :
C6H5CCH + AgNO3+ NH3 C6H5CCAg + NH4NO3


<b>1,00</b>


2. Điều chế :


NO<sub>2</sub>


+ HONO<sub>2</sub>
H2SO4


+ Cl<sub>2</sub>
Fe


NO<sub>2</sub>



Cl
(a)


Cl


+ HONO2


H<sub>2</sub>SO<sub>4</sub>
+ Cl<sub>2</sub>


Fe


Cl


(b) +H2SO4


SO<sub>3</sub>H


Cl


SO<sub>3</sub>H


t


Cl


NO<sub>2</sub>


NO<sub>2</sub>



CH<sub>3</sub>


+ CH3Cl


AlCl<sub>3</sub>


+ Br2


Fe


COOH


(c) + KMnO4


COOH


Br
CH<sub>3</sub>


+ Br2


Fe
+CH<sub>3</sub>Cl


AlCl3


CH<sub>3</sub>


(d) +H2SO4



SO<sub>3</sub>H


CH<sub>3</sub>


SO<sub>3</sub>H


t


CH<sub>3</sub>


Br


Br


+ KMnO4


COOH
Br


<b>0,25</b>


<b>0,50</b>


<b>0,25</b>


<b>0,50</b>


3. (a) X (CxHy), có 12x + y = 128 (y  2x + 2) có hai nghiệm thích hợp là C10H8và C9H20. Tuy nhiên, vì
X tác dụng được với hidro, nên cơng thức đúng là C10H8(7).



Vì X không làm nhạt màu nước brom nên cấu tạo thích hợp của X là naphtalen và phù hợp với
giả thiết thì Y là tetralin và Z là decalin :


</div>
<span class='text_page_counter'>(37)</span><div class='page_container' data-page=37>

(b) Phản ứng :


+ HONO<sub>2</sub> H2SO4 + H<sub>2</sub>O
NO<sub>2</sub>


+ Br<sub>2</sub> Fe + HBr


Fe


</div>
<span class='text_page_counter'>(38)</span><div class='page_container' data-page=38>

<b>ĐỀ SỐ 6</b>


<b>SỞ GIÁO DỤC VÀ ĐÀO TẠO</b>
<b>THÀNH PHỐ ĐÀ NẴNG</b>


<b>HƯỚNG DẪN CHẤM KÌ THI HỌC SINH GIỎI THÀNH PHỐ</b>
<b>NĂM HỌC 2004 - 2005</b>


<b>MƠN: HĨA HỌC LỚP 12 - BẢNG A</b>


Thời gian: 180 phút (không kể thời gian giao đề)


<b>Câu I</b> (3 điểm)


<b>1.</b> Tỉ lệ triti so với tổng số nguyên tử hidro trong một mẫu nước sông là 8.10-18. Triti phân hủy phóng xạ với chu kì
bán hủy 12,3 năm. Có bao nhiêu nguyên tử triti trong 10gam mẫu nước sông trên sau 40 năm.



<b>2.</b> (a) Gọi tên và viết đồng phân cho các hợp chất phức [Pt(NH3)4Cl2]2+và [Co(en)2Cl2].H2O.


(b) Giải thích sự hình thành liên kết trong [CoF6]3-(chất thuận từ) và [Co(CN)6]3-(chất nghịch từ) theo thuyết
liên kết hóa trị VB.


<b>3.</b> Viết phương trình phản ứng minh họa quá trình điều chế các chất sau đây từ các đơn chất halogen tương
ứng: (a) HClO4, (b) I2O5, (c) Cl2O, (d) OF2, (e) BrO3.


<b>Đáp Án</b> <b>Điểm</b>


<b>1.</b> Số nguyên tử 3<sub>1</sub>T trong 10g mẫu = 2 6,02.1023 8.10 18
18


10 <sub></sub>





 = 5,35.106<sub>nguyên tử</sub>


Từ kt


n
n
ln


o


 và



2
/
1
t


693
,
0
k 


 số nguyên tử 3<sub>1</sub>T còn lại sau 40 năm:
40


3
,
12


693
,
0
6


e
10
.
35
,
5
n








 = 5,8.105<sub>nguyên tử</sub>


T
3


1


<b>2.</b> (a) Tên gọi và đồng phân:


[Pt(NH3)4Cl2]2+: ion diclorotatraaminplatin (IV)


[Co(en)2Cl2].H2O: diclorobis(etilendiamin)coban(II) monohidrat


H<sub>3</sub>N


H<sub>3</sub>N NH<sub>3</sub>
NH<sub>3</sub>
Cl


Cl


H<sub>3</sub>N


H<sub>3</sub>N Cl



NH<sub>3</sub>
Cl


NH<sub>3</sub>


trans-


</div>
<span class='text_page_counter'>(39)</span><div class='page_container' data-page=39>

Cl


Cl


Cl
Cl


trans-


cis-en
en


en
en



cis-en
en


(b) Phức [CoF6]3-thuận từ cho thấy các ion F-phối trí vào các obitan lai hóa ngồi sp3d2cịn
trống của Co(III):



F- F- F- F- F- F


-lai hãa sp3d2


Phức [Co(CN)6]3-nghịch từ cho thấy các ion CN-phối trí vào các obitan lai hóa trong d2sp3
cịn trống của Co(III):


CN- CN-CN- CN
-CN- CN


-lai hãa d2sp3


<b>3.</b> (a) 3Cl2+ 6NaOH



t 5NaCl + NaClO3+ 3H2O
4NaClO3 t NaCl + 3NaClO4


NaClO4+ H2SO4NaHSO4+ HClO4(chưng cất)
(b) 3I2+ 6OH-5I-+ ClO3-+ 3H2O


IO3-+ H+HIO3
2HIO3



t I2O5+ H2O
(c) 2Cl2+ HgOCl2O + HgCl2
(d) 2F2+ 2OH-2F-+ OF2+ H2O
(e) Br2+ 2O3  0C


o


2BrO3


<b>Câu II</b> (4 điểm)



<b>1.</b> Tính khối lượng amoniclorua và thể tích dung dịch natri hidroxit 3,0M cần thêm vào 200mL nước và sau đó
pha lỗng đến 500mL để điều chế dung dịch đệm có pH = 9,5 với nồng độ muối là 0,1M. Biết pKb(NH3) = 4,76.


</div>
<span class='text_page_counter'>(40)</span><div class='page_container' data-page=40>

nhau bằng cầu muối. Nối các điện cực với nhau qua một Vol kế. Giả thiết rằng H2SO4điện li hồn tồn và thể
tích dung dịch trong mỗi cốc bằng nhau. Cho Eo 0,771V


Fe
/


Fe3 2  và E 1,51V


o
Mn
/
H
,
nO


M <sub>4</sub>  2  .


(a) Tính thế mỗi điện cực trước khi phản ứng xảy ra. Viết sơ đồ pin và tính hiệu thế pin.
(b) Tính hằng số cân bằng và thế mỗi điện cực tại cân bằng.


<b>3.</b> Cho 1L dung dịch HClO40,003M chứa 2,0.10-4mol mỗi ion Mn2+và Cu2+. Bão hòa dung dịch này bằng H2S, giả
thiết nồng độ H2S luôn bằng 0,1M khơng phụ thuộc sự có mặt của các chất khác.


(a) Hãy cho biết có thể tách riêng hai ion Mn2+<sub>và Cu</sub>2+<sub>khơng? Biết H</sub>


2S có pK1= 7 và pK2= 14. TMnS= 2,5.10-10
và TCuS= 8,5.10-36.



(b) Kết luận trên có thay đổi khơng nếu thay HClO4bằng dung dịch đệm có [H+] = 10-9M?


<b>Đáp Án</b> <b>Điểm</b>


<b>1.</b> Từ
]
NH
[
]
NH
[
lg
pK
pH
4
3


a  <sub></sub>


 
]
NH
[
]
NH
[
lg
)
76


,
4
14
(
5
,
9
4
3




  1,8


]
NH
[
]
NH
[
4
3 <sub></sub>


Trong dung dịch cuối:

 

NH<sub>4</sub> 0,1M nên

NH<sub>3</sub>

1,80,1M0,18M
 n (0,1mol.L 1) (0,5L) 0,05mol


NH<sub>4</sub>   




 và
mol
09
,
0
)
L
5
,
0
(
)
L
.
mol
18
,
0
(


n<sub>NH</sub> 1


3   




Từ phản ứng NH4++ OH-NH3+ H2O ta thấy rằng số mol NaOH cần dùng bằng số mol NH3và
tổng lượng muối amoni cần ban đầu là 0,05mol + 0,09mol = 0,14mol.



 0,03L


)
L
.
mol
0
,
3
(
)
mol
09
,
0
(
V
1


ddNaOH  <sub></sub>  , mNH4Cl (0,14mol)(53,5g/mol)7,49g


<b>2.</b> (a) 0,671V


15
,
0
003
,
0
lg


059
,
0
771
,
0
]
Fe
[
]
Fe
[
lg
059
,
0
E
E <sub>2</sub>
3
o
Fe
/
Fe


Fe  3 2  <sub></sub>   





V


52
,
1
005
,
0
)
1
)(
02
,
0
(
lg
5
059
,
0
51
,
1
]
Mn
[
]
H
[
]
MnO
[

lg
5
059
,
0
E
E
8
2
8
4
o
Mn
/
nO
M
Mn 2
4




    <sub></sub> 


Sơ đồ pin:


PtFe2+<sub>(0,150M), Fe</sub>3+<sub>(0,003M)</sub><sub></sub><sub>MnO</sub>


4-(0,02M), Mn2+(0,005M), H+(1M)Pt
Hiệu thế pin:



Epin= EMn- EFe= 1,52V - 0,671V = 0,85V


(b) Hằng số cân bằng:


63
,
62
059
,
0
)
771
,
0
51
,
1
.(
5
059
,
0
E
n
10
10
10


K   



</div>
<span class='text_page_counter'>(41)</span><div class='page_container' data-page=41>

Vì hằng số cân bằng lớn nên phản ứng sau được coi là hoàn toàn:
5Fe2+ <sub>+ MnO</sub>


4-+ 8H+5Fe3++ Mn2++ 4H2O


0,15 0,02 0,003


0,1 0,02 0,1
0,05 0 0,103
Tại cân bằng:


V
79
,
0
05
,
0


103
,
0
lg
059
,
0
771
,
0


]
Fe
[


]
Fe
[
lg
059
,
0
E


E


E <sub>2</sub>


3
o


Fe
/
Fe
Fe


Mn   3 2  <sub></sub>   








<b>3.</b> (a) Coi sự đóng góp H+của H2S là khơng đáng kể ta có [H+] = 0,003M


Từ 21


2
1
2


2
2


10
K
.
K
]
S
H
[


]
S
[
]
H


[   <sub></sub> <sub></sub>



 [S2-<sub>] =</sub>


0,003

1,1.10 M


1
,
0


10 17


2


21 


 <sub></sub>


[Cu2+<sub>][S</sub>2-<sub>] = (2.10</sub>-4<sub>)(1,1.10</sub>-17<sub>) = 2,2.10</sub>-21<sub>> T</sub>


CuS= 8,5.10-36


 Cu2+<sub>kết tủa dưới dạng CuS</sub>


[Mn2+<sub>][S</sub>2-<sub>] = (2.10</sub>-4<sub>)(1,1.10</sub>-17<sub>) = 2,2.10</sub>-21<sub>< T</sub>


MnS= 2,5.10-10


 Mn2+<sub>không kết tủa dưới dạng muối sunfua.</sub>


Nồng độ S2-<sub>thay đổi không đáng kể khi Cu</sub>2+<sub>kết tủa hết, nên khi đó Mn</sub>2+<sub>vẫn chưa kết tủa.</sub>
Vậy có thể tách riêng hai ion này.



(b) [S2-<sub>] =</sub>


 

0 M


10
1
,
0


10 <sub>2</sub> 4


9


21 




 <sub></sub>


 [Mn2+<sub>][S</sub>2-<sub>] = (2.10</sub>-4<sub>)(10</sub>-4<sub>) = 2.10</sub>-8<sub>> T</sub>


MnS= 2,5.10-10


</div>
<span class='text_page_counter'>(42)</span><div class='page_container' data-page=42>

<b>Câu III</b> (3 điểm)


<b>1.</b> Tính G và nhận xét chiều tự xảy ra cho phản ứng: H2O (l)H+(aq) + OH-(aq)
tại 25o<sub>C trong mỗi điều kiện sau:</sub>


(a) [H+] = 10-3M và [OH-] = 10-4M


(b) [H+] = 10-12M và [OH-] = 2.10-8M


<b>2.</b> Hằng số cân bằng (KC) của một phản ứng hóa hợp:


A(k) + B (k)AB (k)
là 1,8. 103<sub>L.mol</sub>–1<sub>tại 25</sub>o<sub>C và 3,45.10</sub>3<sub>L.mol</sub>–1<sub>tại 40</sub>o<sub>C.</sub>


(a) Giả sử H khơng phụ thuộc nhiệt độ, hãy tính Hovà So


(b) Hãy tính các hằng số cân bằng KPvà KXtại 298K và áp suất toàn phần là 1 atm.


<b>3.</b> (a) Thiết lập biểu thức xác định chu kì bán hủy phản ứng một chiều bậc hai dạng Asản phẩm


(b) Tính chu kì phản ứng bậc hai: buta-1,3-dien (k)xiclobuten (k). Biết nồng độ ban đầu của butadien là
0,272M; tại 500K hằng số vận tốc phản ứng là 0,0143M-1<sub>s</sub>-1<sub>.</sub>


<b>Đáp Án</b> <b>Điểm</b>


<b>1.</b> Tại cân bằng ta có: [H+<sub>] = [OH</sub>-<sub>] = 10</sub>-7<sub>M và K</sub>
N= 10-14


 1 1 14 4 1


N
o
mol
.
J
10
.


8
)
10
ln(
)
K
298
)(
mol
.
K
.
J
314
,
8
(
K
ln
RT


G       




(a) GGoRTln([H][OH])


1
4
4


3
1
1
1


4<sub>J</sub><sub>.</sub><sub>mol</sub> <sub>(</sub><sub>8</sub><sub>,</sub><sub>314</sub><sub>J</sub><sub>.</sub><sub>K</sub> <sub>.</sub><sub>mol</sub> <sub>)(</sub><sub>298</sub><sub>K</sub><sub>)</sub><sub>ln(</sub><sub>10</sub> <sub>10</sub> <sub>)</sub> <sub>4</sub><sub>.</sub><sub>10</sub> <sub>J</sub><sub>.</sub><sub>mol</sub>
10


.
8


G         




Vì G0 nên phản ứng không tự xảy ra theo chiều thuận.


(b) 4 1 1 1 12 8 4 1


mol
.
J
10
.
2
,
3
)
10
.


2
10
ln(
)
K
298
)(
mol
.
K
.
J
314
,
8
(
mol
.
J
10
.
8


G         




Vì G0 nên phản ứng tự xảy ra theo chiều thuận.


<b>2.</b> (a) Từ <sub></sub>







 <sub></sub>


2
1
o
1
2
T
1
T
1
R
H
K
K
ln
 




 <sub></sub>



313
1
298
1
314
,
8
H
10
.
8
,
1
10
.
45
,
3
ln
o
3
3


 Ho<sub>= 33,77 kJ/mol</sub>


</div>
<span class='text_page_counter'>(43)</span><div class='page_container' data-page=43>

 1 1
2
2
2
o


o
mol
.
K
.
J
6
,
175
T
K
ln
RT
H


S      




(b) Tại 298K:


1
3


1
C


P 0,726atm


298


.
314
,
8
10
.
8
,
1
)
RT
(
K


K     


726
,
0
P
K
K<sub>x</sub>  <sub>P</sub> 


<b>3.</b> (a) <i>k</i>[A]2=

 



t
A




 <sub></sub>


   

kt
A
1
A
1
o



khi t =
2
1


t thì:

   


2
A


A  0 <sub></sub>


   

0 0

 

0
2
/
1
A
k
1
A
1

2
A
1
k
1
t 















(b)

<sub> </sub>

257s


)
M
272
,
0
)(
s


M
0143
,
0
(
1
A
k
1


t <sub>1</sub> <sub>1</sub>


o
2


1     


<b>Câu IV</b> (3 điểm)


<b>1.</b> Hoàn thành các dãy chuyển hóa dưới đây:


axetilen + CH3MgBrG + CH4
G + CO2H 



H


I (C3H2O2)
I

H

2O

, H

2SO

4,

HgSO

4

J (C



3H4O3)


J + KMnO4sản phẩm chỉ chứa một chất hữu cơ K.


<b>2.</b> So sánh (có giải thích) độ mạnh tính axit của K với (a) axit oxalic và (b) axit sucxinic.


<b>Đáp Án</b> <b>Điểm</b>


<b>1.</b> Hoàn thành dãy chuyển hóa


CHCH + CH3MgBrCHCMgBr + CH4
CHCMgBr + CO2CHCCO2MgBr 




H <sub>CH</sub><sub></sub><sub>CCOOH</sub>
CHCCOOH H2O, H2SO4,HgSO4 HOCCH


2COOH
HOCCH2COOH KMnO4 HOOCCH2COOH


</div>
<span class='text_page_counter'>(44)</span><div class='page_container' data-page=44>

giảm làm độ phân cực của liên kết O-H và giảm độ bền của bazơ liên hợp. K2(axit oxalic) > K2
(axit malonic) do ion oxalat đặc biệt bền nhờ sự cộng hưởng.


(b) K1(axit malonic) > K1(axit sucxinic) do khi mạch cacbon tăng, hiệu ứng (-I) của nhóm cacboxyl
giảm làm độ phân cực của liên kết O-H giảm và độ bền của bazơ liên hợp giảm. K2(axit malonic)
< K2 (axit sucxinic) do ion sucxinat bền hơn ion malonat nhờ mạch cacbon lớn hơn, thế năng
tương tác giữa các nhóm COO-<sub>nhỏ hơn.</sub>


C



O O


C
OH
HO


C


O O


C
O
-HO


C


O O


C
O


O


2-HOOC(CH2)nCOOHHOOC(CH2)nCOO--OOC(CH2)nCOO


<b>-Câu V</b> (3 điểm)


<i>Citral</i>, C10H16O, là một tecpen đóng vai trị cấu tử chính cho dầu cỏ chanh. Nó phản ứng với hidroxylamin sinh ra
một hợp chất có cơng thức C10H7ON, và với thuốc thử Tollens cho kết tủa bạc cùng hợp chất có cơng thức


C10H16O2. Oxi hóa mãnh liệt citral sinh ra axeton, axit oxalic và axit levulinic (CH3COCH2CH2COOH).


<b>1.</b> Đề nghị cấu tạo cho citral phù hợp với các dữ liệu trên và phù hợp với quy tắc isopren.


<b>2.</b> Trong thực tế citral gồm hai đồng phân là citral a và citral b. Oxi hóa êm dịu geraniol sinh ra citral a (<i>geranial</i>),
còn từ nerol thì sinh ra citral b (<i>neral</i>). Trong axit sunfuric, cả geraniol và nerol đều chuyển hóa thành
-tecpineol nhưng khả năng chuyển hóa của geraniol chậm hơn nhiều so với nerol. Xác định cấu trúc phù hợp và
gọi tên theo IUPAC cho hai đồng phân này. Viết cơ chế phản ứng tạo thành-tecpineol từ nerol.


<b>3.</b> Khi cho<i>citral</i>phản ứng với<i>olivetol</i>trong axit loãng thu được một hỗn hợp các sản phẩm trong đó chứa (I) .
Cho biết cơ chế hình thành (I) và gọi tên từng giai đoạn của cơ chế.


OH


HO C<sub>5</sub>H<sub>11</sub><i>-n</i>


Olivetol


OH


C<sub>5</sub>H<sub>11</sub><i>-n</i>
1


-3,4-trans-tetrahidrocannabinol (I)
O


OH
-tecpineol


<b>Đáp Án</b> <b>Điểm</b>



<b>1.</b> <i>Citral</i>C10H16O có độ bất bão hòa 3; Phản ứng với hidroxylamin và thuốc thử Tollens cho
thấy nó là một andehit; Oxi hóa tạo ba phân mảnh cho thấy phân tử có hai liên kết đôi. Xếp các
phân mảnh theo quy tắc isopren ta có cấu tạo của<i>citral</i>:


CH<sub>3</sub> C
CH<sub>3</sub>


OH <sub>O</sub> C


OH


CH<sub>2</sub> CH<sub>2</sub> C OH
CH<sub>3</sub>


O C


OH


C OH


OH


CH<sub>3</sub> C
CH<sub>3</sub>


CH CH<sub>2</sub> CH<sub>2</sub> C
CH<sub>3</sub>


</div>
<span class='text_page_counter'>(45)</span><div class='page_container' data-page=45>

<b>2.</b> Citral a và b là đồng phân hình học liên kết đôi C2: Nerol phản ứng dễ dàng hơn, vậy nerol và



neral là đồng phân<i>cis-</i>(Z) còn geraneol và geranial là đồng phân<i>trans-</i>(E).


C C


H
CH<sub>3</sub>


CH<sub>3</sub> CH<sub>2</sub>
CH<sub>2</sub>


C C


H


CH<sub>3</sub> CHO


C C


H
CH<sub>3</sub>


CH<sub>3</sub> CH<sub>2</sub>
CH<sub>2</sub>


C C


CHO


CH<sub>3</sub> H



Citral a


(E)-2,7-dimetylocta-2,6-dienal (Z)-2,7-dimetylocta-2,6-dienalCitral b
Cơ chế phản ứng (AE):


OH


CH<sub>2</sub>OH CH<sub>2</sub>OH<sub>2</sub>+ CH<sub>2</sub>


+ H+


- H<sub>2</sub>O


+ H<sub>2</sub>O
- H+


<b>3.</b> Cơ chế tạo (I)


HO


HO


C<sub>5</sub>H<sub>11</sub><i>-n</i>
CHO H


+


CHOH olivetol C
OH


H


H+
(3)


HO


HO


C<sub>5</sub>H<sub>11</sub><i>-n</i>
C


H


-H<sub>2</sub>O


HO


HO


C<sub>5</sub>H<sub>11</sub><i>-n</i>


HO


OH


C<sub>5</sub>H<sub>11</sub><i>-n</i> -H


+



(6)


HO


O


C<sub>5</sub>H<sub>11</sub><i>-n</i>


(5)


(4)
(2)


(1)


(1) Proton hóa andehit, (2) Thế SEAr, (3) Proton hóa và tách nước tạo ion dạn alylic, (4) Cộng AE
của cacbocation vào liên kết đôi, (5) Cộng tạo ete đã proton hóa, (6) Tách proton tạo (I).


<b>Câu VI</b> (4 điểm)


<b>1.</b> Người ta xác định cấu tạo chất kháng sinh<i>gramicidin S</i>(chất A) bằng các q trình sau:


 Phân tích sản phẩm thủy phân cho thấy tỉ lệ đẳng phân tử các aminoaxit Leu H3N+CH(<i>i-</i>Bu)COO-(M = 131),
Orn H3N+(CH2)3CH(NH2)COO-(M = 132), Phe PhCH2CH(+NH3)COO-(M = 165), Val H3N+CH(<i>i-</i>Pr)COO-(M =
117), Pro (M = 115). Khối lượng phân tử A khoảng 1300.


N


H H



</div>
<span class='text_page_counter'>(46)</span><div class='page_container' data-page=46>

 Thử nghiệm xác định aminoaxit C-cuối mạch cho kết quả âm; thử ngiệm xác định aminoaxit N-đầu mạch
bằng DNFB chỉ thấy sinh ra DNP-NHCH2CH2CH2CH(+NH3)COO-.


 Thủy phân khơng hồn tồn gramicidin S tạo thành các dipeptit và tripeptit sau: Leu-Phe, Phe-Pro,
Phe-Pro-Val, Val-Orn-Leu, Orn-Leu, Val-Orn, Pro-Val-Orn.


Cho biết trật tự cấu tạo của phân tử gramicidin S.


<b>2.</b> Q trình oxi hóa metyl glycosit Q bằng HIO4tạo sản phẩm giống như sản phẩm oxi hóa metyl α-glycosit của
D-andohexoz. Tuy nhiên, q trình oxi hóa Q chỉ tiêu tốn 1 mol HIO4và khơng tạo ra axit fomic. Metyl hóa Q,
thủy phân, sau đó oxi hóa mãnh liệt thu được axit dicacboxilic di-O-metylete của axit (-)-tartaric. Lập luận viết
cấu hình đầy đủ cho Q (dạng công thức chiếu).


<b>Đáp Án</b> <b>Điểm</b>


<b>1.</b> Từ Mn(131132165115117)1300n = 2


Theo giả thiết<i>gramicidin S</i>khơng có C-cuối mạch. Sản phẩm DNP-NHCH2CH2CH2CH(+NH3)COO
-cho thấy DNFB chỉ phản ứng với

-NH2, nghĩa là<i>gramicidin S</i>cũng khơng có N-đầu mạch. Vậy


<i>gramicidin S</i>có cấu trúc vịng.


Từ kết quả thủy phân khơng hồn tồn: Leu Phe
Phe Pro


Phe Pro Val


Val Orn Leu


Orn Leu


Val Orn


Pro Val Orn


ta có trật tự: Leu-Phe-Pro-Val-Orn-Leu, vậy trật tự cấu tạo của<i>gramicidin S</i> là:
Val Orn Leu Phe


Pro
Val
Orn
Leu
Phe


Pro


<b>2.</b> Cấu hình đầy đủ:


OMe
HO


OH
CH<sub>2</sub>OH


O


OMe
MeO


OMe
CH<sub>2</sub>OH



O


CHO
MeO


OMe
CH<sub>2</sub>OH
OH


COOH
MeO


OMe
COOH
OMe


CHO
CHO
CH<sub>2</sub>OH


O


</div>
<span class='text_page_counter'>(47)</span><div class='page_container' data-page=47>

<b>ĐỀ SỐ 7</b>


<b>SỞ GIÁO DỤC VÀ ĐÀO TẠO</b>
<b>THÀNH PHỐ ĐÀ NẴNG</b>


<b>ĐÁP ÁN KÌ THI HỌC SINH GIỎI THÀNH PHỐ</b>
<b>NĂM HỌC 2004 - 2005</b>



<b>MƠN: HĨA HỌC LỚP 12 - BẢNG B</b>


Thời gian: 180 phút (không kể thời gian giao đề)


<b>Câu VII</b> (3 điểm)


<b>4.</b> Nguyên tử của nguyên tố X có 10 electron thuộc phân lớp p. Thêm đơn chất X hoạt động phóng xạ vào dung
dịch chứa 2


3


XO thu được ion A hoạt động phóng xạ. Thêm dung dịch chứa Ba2+thì thu được kết tủa B. Lọc
tách kết tủa B, sấy khơ rồi xử lí với dung dịch axit clohidric thì thu được chất rắn có hoạt động phóng xạ, chất
khí khơng hoạt động phóng xạ và nước.


(a) Viết các phương trình ion thu gọn minh họa (kí hiệu X*cho ngun tử X hoạt động phóng xạ).


(b) Viết cơng thức cấu tạo cho ion A và cho biết cấu tạo, dạng hình học các hợp khí với H, oxit bậc cao nhất,


hidroxit bậc cao nhất của X.


<b>5.</b> Sản xuất ure từ nguyên liệu đầu là NH3và CO2qua hai công đoạn chính sau:


(a) 2NH3(k) + CO2(k)H2NCOONH4(r) + 159,1kJ (tổng hợp amoni cacbamat)
(b) H2NCOONH4(r)H2NCONH2(r) + H2O (l) - 258kJ (dehidrat hóa tạo ure)
Cho biết các biện pháp có thể áp dụng để làm tăng hiệu suất mỗi quá trình trên.


<b>Đáp Án</b> <b>Điểm</b>



<b>1.</b> (a) Cấu hình electron của X: 1s22s22p63s23p4Z = 16, X là lưu huỳnh (S)
S*<sub>+</sub> 2


3


SO  S*SO2<sub>3</sub> (S<sub>2</sub>O<sub>3</sub>2)
Ba2+<sub>+</sub> 2


3
*


SO


S  BaS*SO<sub>3</sub>


3
*


SO


BaS + 2H+SO2+ S*+ Ba2++ H2O
(b) Cấu tạo của A:


O
S
S


OO





2-*


Vì SO2khơng có hoạt tính phóng xạ nên S*chỉ tham gia liên kết S-S, mà không tham gia
liên kết S-O.


</div>
<span class='text_page_counter'>(48)</span><div class='page_container' data-page=48>

HO
HO


S
H


S
H


S
O


O
O


OO


dạ ng chữ V dạ ng tam giá c phẳng dạ ng tứ diện


<b>2.</b> (a) Dựng d NH3hoc CO2(thực tế chỉ dùng dư NH3), tăng áp suất và hạ nhiệt độ.
(b) Đun nóng (khơng q cao nếu khơng sẽ gây phân hủy ure) và chưng cất nước.


<b>Câu VIII</b> (4 điểm)



<b>4.</b> Mỗi hỗn hợp gồm hai chất sau đây có thể tồn tại được hay khơng ? Nếu có tồn tại thì hãy cho biết điều kiện,
nếu khơng tồn tại thì giải thích rõ ngun nhân: (a) H2và O2, (b) SO2và NO2, (c) Na2O2và H2O, (d) dd FeCl2và
Br2, (e) dd FeCl3và KI


<b>5.</b> X là hợp chất hoá học tạo ra trong hợp kim gồm Fe và C trong đó có 6,67% C về khối lượng. Hồ tan X trong
HNO3đặc nóng thu được dung dịch A và hỗn hợp khí B. Cho A, B lần lượt tác dụng với NaOH dư thì A tạo kết
tủa A1, B tạo hỗn hợp B1có 3 muối. Nung A1và B1ở nhiệt độ cao A1tạo oxit A2, B1tạo hỗn hợp B2gồm hai
muối. Cho B2tác dụng với H2SO4 lỗng thu được khí B3 và axit B4. Chất B4làm mất màu dung dịch KMnO4
(trong môi trường axit). Viết các phương trình phản ứng (các phản ứng trong dung dịch viết dạng ion thu gọn).
<b>6.</b> Tính thể tích amoniac đậm đặc (14,8M) và khối lượng amoni clorua cần dùng để pha chế 100mL dung dịch


đệm có pH = 10, nếu nồng độ của muối là 0,2M. Biết pKb(NH3) = 4,76.


<b>Đáp Án</b> <b>Điểm</b>


<b>1.</b> (a) Tồn tại ở điều kiện thường, phản ứng khi đun nóng : H2+
2
1


O2 
o


t <sub>H</sub>


2O.


(b) Không : SO2+ NO2SO3+ NO


(c) Không: Na2O2+ H2O2NaOH +
2


1


O2
(d) Không : 6FeCl2+ 3Br24FeCl3+ 2FeBr3
(e) Không : FeCl3+ KI2FeCl2+ 2KCl + I2


<b>2.</b> Với FexCy, ta có 3
y
x
67
,
6


33
,
93
y
12


x
56





 , cơng thức Fe3C
Fe3C + 22H++ 13NO3-3Fe3++ CO2+ 13NO2+ 11H2O
Fe3+<sub>+ 3OH</sub>-<sub></sub><sub>Fe(OH)</sub>


</div>
<span class='text_page_counter'>(49)</span><div class='page_container' data-page=49>

CO2+ 2OH-CO32-+ H2O


2NO2+ 2OH-NO2-+ NO3-+ H2O
2Fe(OH)3 t Fe2O3+ 3H2O
NaNO3 t NaNO2+ 1/2O2
CO32-+ 2H+CO2+ H2O
NO2-+ H+HNO2


5HNO2+ 2MnO4-+ H+5NO3-+ 2Mn2++ 3H2O
A: Fe(NO3)3 B: CO2, NO2


A1: Fe(OH)3 B1: Na2CO3, NaNO3, NaNO2
A2: Fe2O3 B2: Na2CO3, NaNO2


B3: CO2
B4: HNO2


<b>3.</b> m<sub>NH</sub><sub>Cl</sub> (0,1L) (0,2mol.L 1) (53,5g.mol 1) 1,07g


4    







 









4
3
b


NH
NH
lg
)
pK
14
(


pH , 10 = (14 - 4,76) +



1
3


L
.
mol
2
,
0


NH


lg <sub></sub>


 [NH3] = 1,15 (mol.L-1), nNH<sub>3</sub> (1,15mol.L 1)(0,1L)0,115mol





 7,8.10 L


L
.
mol
8
,
14


mol
115
,
0


V 3


1
ddNH<sub>3</sub>




 




<b>Câu IX</b> (3 điểm)



Cho 2,16gam hỗn hợp gồm Al và Mg tan hết trong dung dịch axit HNO3lỗng, đun nóng nhẹ tạo ra dung dịch A và
448mL (đo ở 354,9K và 988mmHg) hỗn hợp khí B khơ gồm 2 khí khơng màu, khơng đổi màu trong khơng khí. Tỷ
khối của B so với oxi bằng 0,716 lần tỷ khối của CO2so với nitơ. Làm khan A một cách cẩn thận thu được chất rắn
D, nung D đến khối lượng không đổi thu được 3,84 gam chất rắn E. Viết phương trình phản ứng, tính lượng chất D
và % lượng mỗi kim loại trong hỗn hợp ban đầu.


<b>Đáp Án</b> <b>Điểm</b>


i. <sub>Phương trình phản ứng: Khí B theo giả thiết chứa N2 và N2O.</sub>
5Mg + 12H+<sub>+ 2NO</sub>


3 5Mg
2+<sub>+ N</sub>


</div>
<span class='text_page_counter'>(50)</span><div class='page_container' data-page=50>

4Mg + 10H+<sub>+ 2NO</sub>


3 4Mg
2+<sub>+ N</sub>


2O+ 5H2O
10Al + 36H+<sub>+ 6NO</sub>


3


 <sub></sub>


10Al3+<sub>+ 3N</sub>


2+ 18H2O
8Al + 30H+<sub>+ 6NO</sub>



3




8Al3+<sub>+ 3N</sub>


2O+ 15H2O
4Al(NO3)32Al2O3+ 12NO2+ 3O2
2Mg(NO2)2 2MgO + 4NO2+ O2


Với KL mol TB của 2 khí = 36 và tổng số mol 2 khí = 0,02 ta có thể tính được:
Số mol N2 = 0,01 mol và số mol N2O = 0,01mol


Al – 3e  Al3


2N5




+ 10 e <sub>N2.</sub>
x 3x 0,1 0,01
Mg – 2e  Mg2




2N5





+ 8 e  <sub>N2O</sub>
y 2y 0,08 0,01


Bảo tồn electron ta có: 3x + 2y = 0,18 và 27x + 24y = 2,16x = 0.


Từ đây nảy sinh tình huống có vấn đề? Theo định luật bảo toàn khối lượng: 3,84 gam chất E chắc
chắn là Al2O3 và MgO. Từ lượng 2 kim loại và lượng 2 oxit tính được số mol Al = 0,04 và số mol Mg
= 0,045. Lặp lại tính toán như trên :


Al – 3e Al3


2N5




+ 10 e N2
0,04 0,12 0,1 0,01
Mg – 2e Mg2




2N5




+ 8 e  N2O
0,045 0,09 0,08 0,01



ta thấy : tổng số mol e nhường (0,21) > tổng số mol e thu (0,18)chứng tỏ còn một phần N5




= 0,21 –
0,18 = 0,03 mol đã tham gia phản ứng khác, khơng giải phóng khí.


Đó là phản ứng: 4Mg + 10H+<sub>+ NO</sub>


3


 <sub></sub>


4Mg2+<sub>+ NH</sub>


4




+ 3H2O
8Al + 30H+<sub>+ 3NO</sub>


3


 <sub></sub>


8Al3+<sub>+ 3NH</sub>


4





+ 9H2O
2NH4NO3N2+ O2+ 4H2O


</div>
<span class='text_page_counter'>(51)</span><div class='page_container' data-page=51>

gam. Hỗn hợp ban đầu có 50% khối lượng mỗi kim loại.


<b>Câu X</b>(3 điểm)


<b>3.</b> Cho 6 dung dịch : glucozơ, axit fomic, axit axetic, andehit axetic, etylenglicol và rượu etylic. Phân biệt 6 dung


dịch trên bằng phương pháp hoá học và viết sơ đồ phản ứng chuyển hoá glucozơ thành 5 chất cịn lại.


<b>4.</b> Trong các dãy chuyển hóa điều chế axit picric từ benzen dưới đây, dãy nào hợp lí và dãy nào khơng hợp lí. Giải


thích rõ sự hợp lí và khơng hợp lí đó.
(a) benzen



(1)


phenyl clorua



(2)


phenol



(3)


axit picric


(b) benzen (1) <sub>phenyl clorua</sub> <sub></sub><sub></sub>(2) <sub>1-clo-2,4-dinitrobenzen</sub> <sub></sub><sub></sub>(3) <sub>2,4-dinitrophenol</sub> <sub></sub><sub></sub>(4) <sub>axit</sub>
picric


<b>Đáp Án</b> <b>Điểm</b>


<b>1.</b> Tóm tắt cách giải:



glucozơ axit fomic axit axetic and axetic etylenglicol rượu
etylic


q tím - (B) đỏ (A) đỏ (A) - (B) - (B) - (B)


(A) AgNO3
trong NH3


kết tủa
trắng bạc




-(B) Cu(OH)2


trong OH-<sub>, t</sub> dd xanh


đỏ gạch


đỏ gạch dd xanh


-C6H12O6

men

 

C2H5OH

CuO

 

,t CH3CHO 

2
2,Mn


O <sub>CH</sub>


3COOH


C2H5OH H2SO4,t CH2=CH2 KMnO4CH2(OH)CH2(OH)


CH3COOH

NaOH

,CaO

,t

CH4  
o
600
400
,
NO


HCHO 2
2,Mn
O


HCOOH


<b>2.</b> (a) benzen (1) phenyl clorua (2) phenol (3) axit picric


</div>
<span class='text_page_counter'>(52)</span><div class='page_container' data-page=52>

(b) benzen



(1)


phenyl clorua



(2)


1-clo-2,4-dinitrobenzen



(3)


2,4-dinitrophenol



(4)


axit picric



Dãy này hợp lí. Giai đoạn (2) do Cl- và -NO2đều là nhóm phản hoạt hóa nên chỉ thế dinitro. Giai
đoạn (4), hai nhóm -NO2làm bền hóa phenol, tránh được q trình oxi hóa xảy ra khi nitro hóa.


<b>Câu XI</b> (4 điểm)


<b>4.</b> Hidrocacbon mạch thẳng A có m<sub>C</sub>:m<sub>H</sub> 36:7. Xác định cơng thức cấu tạo A và hồn thành dãy chuyển
hố:


AA2A3Phenol H2,Ni A4CuOA5 


H
,
KMnO<sub>4</sub>


A6 tơ nilon 6,6


<b>5.</b> Tiến hành thí nghiệm như sau: Cho 1mL CHCl3đã rửa sạch ion halogenua vào ống nghiệm có sẵn 3mL dung
dịch NaOH 10% trong ống nghiệm, lắc đều và đun sôi hỗn hợp một cách cẩn thận. Làm lạnh hỗn hợp phản
ứng, gạn lấy phần dung dịch trong ở phía trên rồi chia làm ba phần:


Phần 1: Cho thêm vài giọt dung dịch HNO3, sau đó thêm tiếp vài giọt dung dịch AgNO31% .
Phần 2: Cho 1mL dung dịch AgNO3/NH3vào phần 2 rồi đun nóng nhẹ.


Phần 3: Cho vài giọt dung dịch KMnO41% vào phần 3.


Nêu hiện tượng xảy ra trong 3 thí nghiệm ở 3 phần dung dịch trên, giải thích, viết phương trình phản ứng
minh họa.


<b>6.</b> Viết cơ chế phản ứng dehidrat hóa 3,3-dimetylbutan-2-ol và xiclobutylmetanol trong H2SO4đặc.



<b>Đáp Án</b> <b>Điểm</b>


(a) 3:7


1
7
:
12
36
n
:


</div>
<span class='text_page_counter'>(53)</span><div class='page_container' data-page=53>

Công thức nguyên: (C3H7)n


Ta có điều kiện: 7n 6n + 2 và 7n là số chẵn
 n = 2, công thức phân tử của A C6H14
Cơng thức cấu tạo: CH3(CH2)4CH3(<i>n-</i>hexan)
Dãy chuyển hóa:


Cl OH OH


O


<i>n-C</i><sub>6</sub>H<sub>14</sub> Al2O3/Cr2O3 Cl2, as <sub>425</sub>H2Oo


H2, Ni


CuO, t KMnO4, H+



HOOC(CH<sub>2</sub>)<sub>4</sub>COOH


H<sub>2</sub>N(CH<sub>2</sub>)<sub>6</sub>NH<sub>2</sub>


T¬ nilon-6,6


(b) CHCl3 + 3 NaOH 
0


<i>t</i> <sub>HCOONa + 3 NaCl + 2 H</sub>
2O
Ở phần 1 xuất hiện kết tủa trắng do :


NaCl + AgNO3 AgCl + NaNO3


Ở phần 2 xuất hiện kết tủa Ag ( phản ứng tráng gương) do :
HCOONa + 2 [Ag(NH3)2]OH 


0


<i>t</i>


NaHCO3 + 2Ag + 4NH3+ H2O


Ở phần 3 , dung dịch màu tím hồng của KMnO4chuyển sang màu xanh lá cây do:
HCOONa + 2 KMnO4+ 3 NaOH Na2CO3 + K2MnO4+ Na2MnO4+ 2 H2O


(c) Cơ chế:


CH<sub>3</sub> C


CH<sub>3</sub>
CH<sub>3</sub>


CH
OH


CH<sub>3</sub> CH<sub>3</sub> C


CH<sub>3</sub>
CH<sub>3</sub>


CH
OH<sub>2</sub>


CH<sub>3</sub> CH<sub>3</sub> C


CH<sub>3</sub>
CH<sub>3</sub>


CH CH<sub>3</sub>


CH<sub>3</sub> C
CH<sub>3</sub>


CH CH<sub>3</sub>
CH<sub>3</sub>


H+


- H2O



-H+ CH3 C


CH<sub>3</sub>


</div>
<span class='text_page_counter'>(54)</span><div class='page_container' data-page=54>

CH<sub>2</sub>OH H CH<sub>2</sub>OH<sub>2</sub>


+


- H<sub>2</sub>O <sub>- H</sub>+


<b>Câu XII</b> (3 điểm)


Hỗn hợp Y gồm hai chất hữu cơ A và B cùng chức hố học. Nếu đun nóng 15,7 gam hỗn hợp Y với NaOH dư thì thu
được muối của một axit hữu cơ đơn chức và 7,6 gam hỗn hợp hai rượu no đơn chức bậc nhất kế tiếp nhau trong
cùng dãy đồng đẳng. Nếu đốt 15,7 gam hỗn hợp Y thì cần dùng vừa hết 21,84 lít O2và thu được 17,92 lít CO2(các thể
tích khí đo ở điều kiện tiêu chuẩn). Xácđịnh công thức cấu tạo của A và B.


<b>Đáp Án</b> <b>Điểm</b>


Este đơn chức kế tiếp có dạng chung CxHyO2
mol


975
,
0
n


2



O  ; nCO<sub>2</sub> 0,8mol


 0,65mol


18


44
8
,
0
32
975
,
0
7
,
15
n<sub>H</sub><sub>O</sub>


2 









 0,15mol



2


2
975
,
0
2
8
,
0
65
,
0


n<sub>Y</sub>      


 5,33


15
,
0


8
,
0


C  (5 < 5,33 < 6)


66
,


8
2
15
,
0


65
,
0


H   (8 < 8,66 < 10)
 Công thức phân tử 2 este là C5H8O2và C6H10O2


66
,
50
15
,
0


6
,
7
MROH  


 Hai rượu kế tiếp là C2H5OH (M = 46) và<i>n-</i>C3H7OH (M = 60)


</div>
<span class='text_page_counter'>(55)</span><div class='page_container' data-page=55>

<b>ĐỀ SỐ 8</b>


<b>SỞ GIÁO DỤC - ĐÀO TẠO</b>


<b>THÀNH PHỐ ĐÀ NẴNG</b>


<b>KÌ THI HỌC SINH GIỎI THÀNH PHỐ</b>
<b>NĂM HỌC 2005 - 2006</b>


<b>HƯỚNG DẪN CHẤM MÔN HỌC LỚP 12 - BẢNG A</b>


<b>Câu 1</b> (3,0 điểm)


1. Dựa vào phương pháp gần đúng Slater, tính năng lượng ion hóa I1cho He (Z = 2).


2. Dựa trên mơ hình VSEPR, giải thích dạng hình học của các phân tử NH3, ClF3và XeF4.


3. Hoạt tính phóng xạ của đồng vị 210<sub>84</sub>Po giảm đi 6,85 % sau 14 ngày. Xác định hằng số tốc độ của quá


trình phân rã, chu kỳ bán hủy và thời gian để cho nó bị phân rã 90 %.


<b>ĐÁP ÁN</b> <b>ĐIỂM</b>


<b>Câu 1</b> (3,0 điểm)


1. He có cấu hình 1s2,

78,6eV
1


3
,
0
2
6
,


13
2
n


Z
6
,
13
2
E


2


2
*


2
*
*


He 












 <sub></sub>















He+<sub>có cấu hình 1s</sub>1<sub>,</sub> <sub>54</sub><sub>,</sub><sub>4</sub><sub>eV</sub>
1


2
6
,
13
n


Z
6
,
13
E



2


2
2
*


He 










Q trình ion hóa He - eHe+<sub>;</sub>


 I E* E*<sub>He</sub> ( 54,4eV) ( 78,6eV) 24,2eV
He


1       


<b>0,75</b>


2. Cấu tạo của phân tử NH3cho thấy quanh N có 4 khơng gian khu trú, trong đó có 1


cặp electron tự do (AB3E) nên phân tử NH3có dạng tháp đáy tam giác với góc liên


kết nhỏ hơn 109o<sub>28</sub>'<sub>(cặp electron tự do địi hỏi một khoảng khơng gian khu trú lớn</sub>



hơn):


N
H


H
H
<b>..</b>


H
N


HH


<b>0,50</b>


Phân tử ClF3có năm khoảng khơng gian khu trú, trong đó có hai cặp electron tự do


(AB3E2) nên có dạng chữ T (các electron tự do chiếm vị trí xích đạo):


</div>
<span class='text_page_counter'>(56)</span><div class='page_container' data-page=56>

Cl
F


F
F
<b>.. ..</b>


F Cl
F



F


Phân tử XeF4có sáu khoảng khơng gia khu trú, trong đó có hai cặp electron tự do


(AB4E2) nên có dạng vng phẳng (các cặp electron tự do phân bố xa nhau nhất):
Xe


F
F
F
F <b>..</b>


<b>..</b>


F
Xe


F F


F


<b>0,50</b>


3. Từ


100 6,85

0,00507


100
ln



14
1
k
kt
m
m
ln


t


0 <sub></sub>







 ngày-1


137
00507
,
0


693
,
0
k



2
ln


t<sub>1</sub><sub>/</sub><sub>2</sub>   


 ngày


Thời gian để 210<sub>84</sub>Po bị phân rã 90% là:


100 90

454


100
ln
00507
,
0


1


t 




 ngày


<b>0,75</b>


<b>Câu 2</b> (3,0 điểm)


1. Trộn 15,00 mL dung dịch CH3COONa 0,030 M với 30,00 mL dung dịch HCOONa 0,15 M. Tính pH của



dung dịch thu được. Biết pKa(CH3COOH) = 4,76 và pKa(HCOOH) = 3,75.


2. Tính nồng độ ban đầu của HSO4-(Ka= 10-2), biết giá trị sức điện động của pin:


PtI-0,1 M; I3-0,02 MMnO4-0,05 M, Mn2+0,01 M, HSO4-C MPt


ở 25oC đo được bằng 0,824 V. Cho Eo 1,51V
Mn


/


MnO4 2  và E 0,5355V
o


I
/


I3   .


3. Tính nồng độ ion H+<sub>đủ để làm giảm nồng độ Ag(NH</sub>


3)2+0,10 M xuống còn 1,0.10-8M. Biết pKb(NH3) =


4,76 và hằng số bền β[Ag(NH3)2+] = 7,24.


<b>ĐÁP ÁN</b> <b>ĐIỂM</b>


<b>Câu 2</b> (3,0 điểm)



1. 0,010M


45
15
03
,
0


C


-3COO


CH 




 ; 0,100M


45
30
15
,
0


C<sub>HCO</sub><sub>O</sub>- 





Các cân bằng:



</div>
<span class='text_page_counter'>(57)</span><div class='page_container' data-page=57>

CH3COO-+ H2O⇌CH3COOH + OH- <i>Kb</i>= 10-9,24 (2)


HCOO-+ H2O⇌HCOOH + OH- <i>Kb</i>’= 10-10,25 (3)


Do 11,25


HCOO
'
b
24
,
11
O
CO
CH


b C 10 K C 10


K


-3




 <sub></sub> <sub></sub> <sub></sub>




  cho nên không thể tính gần



đúng theo một cân bằng.


Điều kiện proton: h = [H+<sub>] = [OH</sub>-<sub>] - [CH</sub>


3COOH] - [HCOOH]


]
O
HCO
[
)
K
(
]
O
CO
CH
[
K
1
K
h

-1
'
a

-3
1


a
w

 <sub></sub>

 (4)


Chấp nhận [CH3COO-]o= 0,010; [HCOO-]o= 0,10 và thay vào (4) để tính h1:
9
1
75
,
3
2
76
,
4
14


1 2,96.10


10
.
10
10
.
10
1
10



h <sub></sub> <sub></sub> 










Từ giá trị h1tính lại [CH3COO-]1và [HCOO-]1theo các biểu thức:
o

-3
9
76
,
4
76
,
4
1


-3 0,010 [CH COO ]


10
.
96
,


2
10
10
010
,
0
]
O
CO
CH
[  

 <sub></sub>  <sub></sub>
o

-9
75
,
3
75
,
3
1

-]
O
HCO
[
10
,

0
10
.
96
,
2
10
10
10
,
0
]
O
HCO
[  

 <sub></sub>  <sub></sub>


Kết quả lập lại.


Vậy h = 2,96.10-9<sub>= 10</sub>-8,53<sub></sub><sub>pH = 8,5</sub>


<b>1,00</b>


2. Ở điện cực phải: MnO4-+ 8H++ 5e⇌Mn2++ 4H2O


</div>
<span class='text_page_counter'>(58)</span><div class='page_container' data-page=58>

01
,
0
]


H
[
05
,
0
lg
5
059
,
0
51
,
1
]
Mn
[
]
H
][
MnO
[
lg
5
059
,
0
E
E
8
2

8
4
o
Mn
/
MnO
p 2
4







  
V
574
,
0
)
1
,
0
(
02
,
0
lg
2

059
,
0
5355
,
0
]
I
[
]
I
[
lg
2
059
,
0
E
E
3
3
3
o
I
3
/
I
t


3    



   <sub></sub>


E


Δ = Ephải- Etrái0,824 = 1,51 +
5
059
,
0


lg(5[H+]8) – 0,574
Suy ra h = [H+] = 0,053 M


Mặt khác từ cân bằng:


H2SO4-⇌H++ SO42- Ka= 10-2


[] C – h h h


Suy ra h C


K
h
K
h
C
h
a
2


a
2






Thay giá trị h = 0,053 và Ka= 1,0.10-2, tính được C 0,334M
4


HSO 


<b>1,00</b>


3. Do [Ag(NH3)2+] = 1,0.10-8M << 0,10 M, suy ra phức bị phân hủy hồn tồn. Hơn nữa


mơi trường axit, nên có thể bỏ qua q trình tạo phức hidroxo của Ag+. Phản ứng
phân hủy phức:


Ag(NH3)2++ 2H+⇌Ag++ 2NH4+ K = 10-7,24.(109,24)2= 1011,24


Co 0,1 C


Co <sub>-</sub> <sub>C – 0,2 0,1</sub> <sub>0,2</sub>


Vì mơi trường axit và 9,24


NH 10



K
4






 nhỏ nên sự phân li của NH4+có thể bỏ qua.


Xét cân bằng:
Ag+<sub>+ 2NH</sub>


4+⇌Ag(NH3)2++ 2H+ K-1= 10-11,24


C 0,1 0,2 C – 0,2
[] 0,1-10-8<sub>0,2 - 2.10</sub>-8 <sub>10</sub>-8 <sub>C-0,2 + 2.10</sub>-8


</div>
<span class='text_page_counter'>(59)</span><div class='page_container' data-page=59>

M
2015
,
0
C
10
)
2
,
0
.(
1
,


0
10
.
)
2
,
0
C


( 11,24


2
8
2



  


<b>Câu 3</b> (4,0 điểm)


1. Cho 100 gam N2ở nhiệt độ 0oC và áp suất 1 atm. Tính nhiệt Q, công W, biến thiên nội năng ΔU và


biến thiên entanpi ΔH trong các biến đổi sau đây được tiến hành thuận nghịch nhiệt động: (a) nung


nóng đẳng tích tới áp suất bằng 1,5 atm; (b) giãn đẳng áp tới thể tích gấp đơi thể tích ban đầu. Chấp
nhận rằng N2là khí lí tưởng và nhiệt dung đẳng áp khơng đổi trong q trình thí nghiệm và bằng 29,1


J.K-1.mol-1.



2. Hãy xác định bậc của phản ứng:


(CH3)3CBr (aq) + H2O (l)(CH3)3COH (aq) + H++ Br


-từ các dữ kiện thực nghiệm sau đây ở 298K:


t, (s) 0 15000 35000 55000 95000 145000


[(CH3)3CBr], (mol.L-1) 0,0380 0,0308 0,0233 0,0176 0,0100 0,00502


3. Cân bằng: N2(k) + 3H2(k)2NH3(k)


sẽ chuyển dịch chiều nào khi (a) thêm Ar vào hỗn hợp cân bằng nhưng giữ cho thể tích không đổi; (b)
thêm Ar vào hỗn hợp cân bằng nhưng giữ cho áp suất không đổi?


<b>ĐÁP ÁN</b> <b>ĐIỂM</b>


<b>Câu 3</b> (4,0 điểm)


1.

(a) Nung nóng đẳng tích tới áp suất bằng 1,5 atm


Vì V = const nên W = 0


2 1

p

2 1

p

2 1



V


V nC T T nC R T T n C R T T


Q


U


Δ         


273,15 273,15 10138,74J


1
5
,
1
314
,
8
1
,
29
28
100
T
T
P
P
R
C


n <sub>1</sub> <sub>1</sub>


1
2



p 






 <sub></sub>












273,15 273,15 14194,05J


1
5
,
1
1
,
29
28
100


T
T
P
P
C
n
T
T
C
n
H


Δ <sub>1</sub> <sub>1</sub>


1
2
p
1


2


p 


</div>
<span class='text_page_counter'>(60)</span><div class='page_container' data-page=60>

(b) Giãn đẳng áp tới thể tích gấp đơi

<sub></sub>











 1 1


1
2
p
1
2
p


p T T


V
V
C
n
T
T
C
n
H
Δ
Q


2.273,15 273,15

28388,09J
1


,
29
28


100 <sub></sub> <sub></sub>




.8,314.273,15 8110,60J


28
100
T
T
V
V
nR
T
T
nR
V
Δ
P


W <sub>1</sub> <sub>1</sub>


1
2
1



2  






 <sub></sub>







J


,


20278


8110


28388


W


Q


U


Δ


<b>0,75</b>


2.

Phương trình động học của phản ứng:


<sub>   </sub>

<sub></sub> 

<sub></sub>





 kH O (CH ) CBr


dt
CBr
)
CH
(
d


v 3 3 <sub>2</sub> <sub>3</sub> <sub>3</sub>


Do nồng độ của chất tan rất nhỏ, nên có thể coi [H2O] const, phương trình trên


trở thành:


<sub> </sub>

<sub></sub>

<sub></sub>




 k(CH ) CBr


dt
CBr
)
CH
(
d


v 3 3



3


3 với k = k’[H


2O]α’= const


<b>0,75</b>


Thế các dữ kiện thực nghiệm vào các biểu thức của k đối với các phản ứng đơn
giản bậc 0, 1, 2 ta được các kết quả sau đây:


α = 0 α = 1 α = 2


   



t
A
A


k 1 o  1

 

<sub> </sub>



1
o
1
A
A
ln
t
1



k

   

<sub>   </sub>



1
o
1
1
o
1
A
A
A
A
t
1
k




4,80.10-7 <sub>1,4.10</sub>-5 <sub>4,10.10</sub>-4


4,20.10-7 <sub>1,4.10</sub>-5 <sub>4,70.10</sub>-4


3,70.10-7 <sub>1,4.10</sub>-5 <sub>5,50.10</sub>-4


2,95.10-7 1,4.10-5 7,80.10-4
2,27.10-7 1,4.10-5 1,19.10-4


Vậy phản ứng có bậc 1 đối với (CH3)3CBr.



</div>
<span class='text_page_counter'>(61)</span><div class='page_container' data-page=61>

3.

(a) Ta có:


2


n
2


3
H
N


2
NH
3


H
N


2
NH
P


RT
V
K
RT


V
n
n



n
P


P
P
K


2
2


3


2
2


3 



















Vì V, KPvà T = const nên const
n


n
n
K


3
H
N


2
NH
n


2
2


3 


 . Như vậy có sự tăng áp suất


của hệ nhưng khơng có sự chuyển dịch cân bằng.


(b)



2


n
P


RT
V
K


K 









Với T, P và Kp= const, khi thêm Ar đã làm V tăng nên Knphải giảm. Sự thêm


agon đã làm cân bằng chuyển dịch theo chiều nghịch (chiều làm phân li NH3).


Chú thích:


Trong trường hợp này, ta khơng thể dựa vào ngun lí Le Chatelier để dự đốn
chiều diễn biến của phản ứng.


<b>0,50</b>


<b>0,50</b>



<b>Câu 4</b> (3,0 điểm)


1.

<i>Streptimidon</i>là một loại kháng sinh có cơng thức sau:


H


NH


H


CHH<sub>3</sub>3C H


OH
H
O


O


O
H


(a) Mơ tả các trung tâm lập thể của streptimidon bằng kí hiệu E, Z và R, S.


(b) Streptimidon có bao nhiêu đồng phân lập thể, trong đó có bao nhiêu <i>đồng phân enan</i>và bao
nhiêu<i>đồng phân dia</i>?


2.

So sánh và giải thích:


</div>
<span class='text_page_counter'>(62)</span><div class='page_container' data-page=62>

<b>ĐÁP ÁN</b> <b>ĐIỂM</b>



<b>Câu 4</b> (3,0 điểm)


1. (a) Streptimidon có ba trung tâm lập thể:


H


NH


H


CHH<sub>3</sub>3C H


OH
H
O


O


O
H


<b>E</b>


<b>S</b>


<b>R</b>


*
*



<b>0,75</b>


(b) Streptimidon có 23= 8 đồng phân lập thể, trong đó streptimidon và đối quang của
nó tạo cặp đồng phân enan, còn sáu đồng phân lập thể còn lại là đồng phân dia của


streptimidon. <b>0,75</b>


2. (a) Trật tự tăng dần nhiệt độ sơi:


O N


H


khơng phân cực có phân cực tạo được kiên kết H liên


phân tử <b>0,75</b>


(b) Trật tự tăng dần độ mạnh tính bazơ:


N NH N


H


N lai hóa sp2 N lai hóa sp3 N lai hóa sp3và cấu trúc
vịng ít gây án ngữ


khơng gian với N


<b>0,75</b>



<b>Câu 5</b> (3,0 điểm)


1.

(-)<i>-Nicotin</i>, ankaloit có trong thuốc lá, có thể được tổng hợp theo cách sau:


axit nicotinic (<i>3-pyridincacboxilic</i>) + SO2Cl2, tonicotinyl clorua (C6H4ONCl)


nicotinyl clorua + C2H5OCH2CH2CH2CdClG (C11H15O2N), một xeton


G + NH3, H2, xúc tácH (C11H18ON2)


H + HBr + nhiệt độ caoI (C9H12N2) + etyl bromua


</div>
<span class='text_page_counter'>(63)</span><div class='page_container' data-page=63>

Xác định cấu trúc của ()-nicotin và hoàn thành các phản ứng trên.


2.

Chất lơi cuốn bạn tình của lồi bướm đêm có thể được tổng hợp bằng cách dưới đây. Hãy cho biết


cấu trúc và tên gọi của hợp chất này. Hoàn thành các phương trình phản ứng.
1-heptin + LiNH2S (C7H11Li)


S + 1-clo-3-brompropanT (C10H17Cl)


T + Mg, sau đó<i>n-</i>C10H21CHO, sau đó H+U (C21H40O)


U + H2, xúc tác LindlarV (C21H42O)


V + CrO3chất lôi cuốn bạn tình (C21H40O)


<b>ĐÁP ÁN</b> <b>ĐIỂM</b>



<b>Câu 5</b> (3,0 điểm)


1. Cấu trúc và phản ứng:


SOCl<sub>2</sub> EtO(CH<sub>2</sub>)<sub>3</sub>CdCl


N


COOH


N


COCl


N
C
O


C
H<sub>2</sub>


CH<sub>2</sub>
CH<sub>2</sub>
EtO


<b>G</b>


Amin bËc 2


NH3, H2



Ni


HBr


<b>I</b>
<b>H</b>


N
CH


C
H<sub>2</sub>


CH<sub>2</sub>
CH<sub>2</sub>
EtO
NH<sub>2</sub>


N
CH


C
H<sub>2</sub>


CH<sub>2</sub>
CH<sub>2</sub>
Br
NH<sub>2</sub>



N


*


N
H
H


(+)-Nicotin
Alkyl hãa


CH3I, OH


-N


*


N
H<sub>3</sub>C


H


<b>1,50</b>


2. <i>n-</i>C5H11CCHLiNH 2 <i>n-</i>C


5H11CCLi (S)










BrCH2CH2CH2Cl <i>n-</i>C


5H11CCCH2CH2CH2Cl (T)





Mg <i><sub>n-</sub></i><sub>C</sub>


</div>
<span class='text_page_counter'>(64)</span><div class='page_container' data-page=64>







 


nC<sub>10</sub>H<sub>21</sub>CHO, H <i><sub>n-</sub></i><sub>C</sub>


5H11CC(CH2)3CH(OH)C10H21-<i>n</i>
(CH<sub>2</sub>)<sub>3</sub>


C
H
C
<i>n-C</i><sub>5</sub>H<sub>11</sub>
H



CH
OH


C<sub>10</sub>H<sub>21</sub><i>-n</i>


H2


Lindlar


(CH<sub>2</sub>)<sub>3</sub>
C


H
C
<i>n-C</i><sub>5</sub>H<sub>11</sub>
H


C
O


C<sub>10</sub>H<sub>21</sub><i>-n</i>


CrO3


(Z)-6-henicosen-11-on


<b>1,50</b>


<b>Câu 6</b> (4,0 điểm)



1.

.(+)<i>-Raffinozơ</i>, một đường khơng khử được tìm thấy trong đường củ cải, có công thức C18H32O16.


Thủy phân trong môi trường axit tạo ra D-fructozơ, D-galactozơ và D-glucozơ. Thủy phân dưới tác
dụng của enzim -galactosidaza tạo ra D-galactozơ và saccarozơ. Thủy phân dưới tác dụng của
invertaza (một enzim phân cắt saccarozơ) tạo thành D-fructozơ và disaccarit<i>melibiozơ</i>. Metyl hóa
raffinozơ, sau đó thủy phân tạo ra 1,3,4,6-tetra-O-metyl-D-fructozơ,
2,3,4,6-tetra-O-metyl-D-galactozơ và 2,3,4-tri-O-metyl-D-glucozơ. Cho biết cấu trúc của raffinozơ và melibiozơ.


2.

Một giải pháp bảo vệ nhóm amin hiệu quả là tiến hành phản ứng axyl hóa bằng anhydrit phtalic để


hình thành một phtalimit có nhóm thế trên nguyên tử N. Nhóm bảo vệ sau đó có thể gỡ bỏ dễ dàng
bằng hidrazin mà không ảnh hưởng đến cầu nối peptit. Áp dụng giải pháp này viết sơ đồ tổng hợp
Gli-Ala và Ala-Gli.


<b>ĐÁP ÁN</b> <b>ĐIỂM</b>


<b>Câu 6</b> (3,0 điểm)


Melibioz¬


Raffinoz¬


O
HO


OH


O
HO


CH<sub>2</sub>OH


OH


O
OH


O
HO
CH<sub>2</sub>


CH<sub>2</sub>OH
HOH<sub>2</sub>C <sub>O</sub>


H
HO


HO


Galactoz¬


Saccaroz¬


Fructoz¬
Glucoz¬


</div>
<span class='text_page_counter'>(65)</span><div class='page_container' data-page=65>

Melibioz¬


<i>6-O-(-D-Galactopyranosyl)-D-glucopyra</i>noz¬



(-anomer)
O


OH


OH


O
HO
CH<sub>2</sub>OH


OH


O
OH


HO
CH<sub>2</sub>


HO <b><sub>0,50</sub></b>


N<sub>2</sub>H<sub>4</sub>


Gly-Ala


+


Phtalhydrazua


Kali phtalimit



+


H<sub>3</sub>NCH<sub>2</sub>C NHCHCOO
-CH<sub>3</sub>


O NH


NH


O
O


NCH<sub>2</sub>C
O


O


NHCHCOO
-O


CH<sub>3</sub>


NCH<sub>2</sub>COCl
O


O


Ala SOCl<sub>2</sub>



NCH<sub>2</sub>COOH
O


O


H+
H2O


NCH<sub>2</sub>COOEt
O


O


BrCH<sub>2</sub>COOEt


NK
O


O


Đ ối vớ i Ala-Gly: bắt đầu vớ i BrCHCOOEt,và sau đó dù ng+H<sub>3</sub>NCH<sub>2</sub>COO
-CH<sub>3</sub>


<b>1,00</b>


</div>

<!--links-->
Nghiên cứu thái độ của khách hàng đối với sự thay đổi kiểu dáng của dòng sản phẩm Wave - hãng Honda Việt Nam
  • 39
  • 1
  • 4
  • Tài liệu bạn tìm kiếm đã sẵn sàng tải về

    Tải bản đầy đủ ngay
    ×